redo ms

Pataasin ang iyong marka sa homework at exams ngayon gamit ang Quizwiz!

. A client diagnosed with bipolar disorder: depressive episode intentionally overdoses on sertraline (Zoloft). Family members report that the client has experienced anorexia, insomnia, and recent job loss. Which nursing diagnosis should a nurse prioritize? 1. Risk for suicide R/T hopelessness 2. Anxiety: severe R/T hyperactivity 3. Imbalanced nutrition: less than body requirements R/T refusal to eat 4. Dysfunctional grieving R/T loss of employment

1

. A client is diagnosed with bipolar disorder: manic episode. Which nursing intervention would be implemented to achieve the outcome of "Client will gain 2 lb by the end of the week?" 1. Provide client with high-calorie finger foods throughout the day. 2. Accompany client to cafeteria to encourage adequate dietary consumption. 3. Initiate total parenteral nutrition to meet dietary needs. 4. Teach the importance of a varied diet to meet nutritional needs. 1. Provide client with high-calorie finger foods throughout the day.

1

A 16-year-old client diagnosed with schizophrenia spectrum disorder experiences command hallucinations to harm others. The client's parents ask a nurse, "Where do the voices come from?" Which is the appropriate nursing response? 1. "Your child has a chemical imbalance of the brain, which leads to altered perceptions." 2. "Your child's hallucinations are caused by medication interactions." 3. "Your child has too little serotonin in the brain, causing delusions and hallucinations." 4. "Your child's abnormal hormonal changes have precipitated auditory hallucinations.

1

A client diagnosed with schizophrenia receives fluphenazine decanoate (Prolixin Decanoate) from a home health nurse. The client refuses medication at one regularly scheduled home visit. Which nursing intervention is ethically appropriate? 1. Allow the client to decline the medication and document the decision. 2. Tell the client that if the medication is refused, hospitalization will occur. 3. Arrange with a relative to add the medication to the client's morning orange juice. 4. Call for help to hold the client down while the injection is administered.

1

A nurse is assessing a pathological gambler. What would differentiate this client's behaviors from the behaviors of a non-pathological gambler? 1. Pathological gamblers have abnormal levels of neurotransmitters, whereas non-pathological gamblers do not. 2. Pathological gambling occurs more commonly among women, whereas non-pathological gambling occurs more commonly among men. 3. Pathological gambling generally runs an acute course, whereas non-pathological gambling runs a chronic course. 4. Pathological gambling is not related to stress relief, whereas non-pathological gambling is related to stress relief

1

A nurse is working with a client diagnosed with SSD. What criteria would differentiate this diagnosis from illness anxiety disorder (IAD)? 1. The client diagnosed with SSD experiences physical symptoms in various body systems, and the client diagnosed with IAD does not. 2. The client diagnosed with SSD experiences a change in the quality of self-awareness, and the client diagnosed with IAD does not. 3. The client diagnosed with SSD disorder has a perceived disturbance in body image or appearance, and the client diagnosed with IAD does not. 4. The client diagnosed with SSD only experiences anxiety about the possibility of illness, and the client diagnosed with IAD does not.

1

Approximately two million American children have experienced the deployment of a parent to Iraq or Afghanistan. How many of these children either lost a parent or have a parent who was wounded in these conflicts? 1. 48,000 2. 26,000 3. 11,000 4. 8,000

1

Without authorization, a nurse administers an extra dose of narcotic tranquilizer to an agitated client. The nurse's coworker observes this action but does nothing for fear of retaliation. What is the ethical interpretation of the coworker's lack of involvement? 1. Taking no action is still considered an unethical action by the coworker. 2. Taking no action releases the coworker from ethical responsibility. 3. Taking no action is advised when potential adverse consequences are foreseen. 4. Taking no action is acceptable, because the coworker is only a bystander.

1

The diagnosis of catatonic disorder associated with another medical condition is made when the client's medical history, physical examination, or laboratory findings provide evidence that symptoms are directly attributed to which of the following? (Select all that apply.) 1. Hyperthyroidism 2. Hypothyroidism 3. Hyperadrenalism 4. Hypoadrenalism 5. Hyperaphia

1, 2, 3, 4

Which of the following nursing statements exemplify the cognitive process that must be completed by a nurse prior to caring for clients diagnosed with a substance-related disorder? (Select all that apply.) 1. "I am easily manipulated and need to work on this prior to caring for these clients." 2. "Because of my father's alcoholism, I need to examine my attitude toward these clients." 3. "I need to review the side effects of the medications used in the withdrawal process." 4. "I'll need to set boundaries to maintain a therapeutic relationship." 5. "I need to take charge when dealing with clients diagnosed with substance disorders."

1, 2, 4

A client is diagnosed with functional neurological symptom disorder (FNSD). Which of the following symptoms is the client most likely to exhibit? (Select all that apply.) 1. Anosmia 2. Anhedonia 3. Akinesia 4. Aphonia 5. Amnesia

1, 3, 4

A client is exhibiting symptoms of generalized amnesia. Which of the following questions should the nurse ask to confirm this diagnosis? (Select all that apply.) 1. "Have you taken any new medications recently?" 2. "Have you recently traveled away from home?" 3. "Have you recently experienced any traumatic event?" 4. "Have you ever felt detached from your environment?" 5. "Have you had any history of memory problems?"

1, 3, 5

After disturbing the peace, an aggressive, disoriented, unkempt, homeless individual is escorted to an emergency department. The client threatens suicide. Which of the following criteria would enable a physician to consider involuntary commitment? (Select all that apply.) 1. Being dangerous to others 2. Being homeless 3. Being disruptive to the community 4. Being gravely disabled and unable to meet basic needs 5. Being suicidal

1, 4, 5

Which of the following are included in the U.S. Departments of Agriculture and Health and Human Services guidelines to promote health and prevent disease. (Select all that apply.) 1. Increase physical activity and reduce time spent in sedentary behaviors. 2. Limit total calorie intake to 2,000 mg per day. 3. Reduce daily sodium intake to 3,000 mg per day. 4. Consume less than 10 percent of calories from saturated fatty acids. 5. Limit alcohol consumption to one drink per day for women and two drinks per day for men.

1, 4, 5

Which of the following nursing statements and/or questions represent appropriate communication to assess an individual in crisis? (Select all that apply.) 1. "Tell me what happened." 2. "What coping methods have you used, and did they work?" 3. "Describe to me what your life was like before this happened." 4. "Let's focus on the current problem." 5. "I'll assist you in selecting functional coping strategies."

1,2,3

Which of the following interventions should a nurse use when caring for an inpatient client who expresses anger inappropriately? (Select all that apply.) 1. Maintain a calm demeanor. 2. Clearly delineate the consequences of the behavior. 3. Use therapeutic touch to convey empathy. 4. Set limits on the behavior. 5. Teach the client to avoid "I" statements related to expression of feelings.

1,2,4

Which of the following are behavior assessment categories in the Broset Violence Checklist? (Select all that apply.) 1. Confusion 2. Paranoia 3. Boisterousness 4. Panic 5. Irritability

1,3,5

A client diagnosed with bipolar I disorder: manic episode refuses to take lithium carbonate (Lithobid) because of excessive weight gain. In order to increase adherence, which medication should a nurse anticipate that a physician may prescribe? 1. Sertraline (Zoloft) 2. Valproic acid (Depakote) 3. Trazodone (Desyrel) 4. Paroxetine (Pax

2

A client diagnosed with bipolar disorder is distraught over insomnia experienced over the last 3 nights and a 12-pound weight loss over the past 2 weeks. Which should be this client's priority nursing diagnosis? 1. Knowledge deficit R/T bipolar disorder AEB concern about symptoms 2. Altered nutrition: less than body requirements R/T hyperactivity AEB weight loss 3. Risk for suicide R/T powerlessness AEB insomnia and anorexia 4. Altered sleep patterns R/T mania AEB insomnia for the past 3 nights

2

A client has been assigned an admission diagnosis of brief psychotic disorder. Which assessment information would alert the nurse to question this diagnosis? 1. The client has experienced impaired reality testing for a 24-hour period. 2. The client has experienced auditory hallucinations for the past 3 hours. 3. The client has experienced bizarre behavior for 1 day. 4. The client has experienced confusion for 3 weeks.

2

A client has been extremely nervous ever since a person died as a result of the client's drunk driving. When assessing for the diagnosis of AD, within what time frame should the nurse expect the client to exhibit symptoms? 1. To meet the DSM-5 criteria for adjustment disorder, the client should exhibit symptoms within one year of the accident. 2. To meet the DSM-5 criteria for adjustment disorder, the client should exhibit symptoms within three months of the accident. 3. To meet the DSM-5 criteria for adjustment disorder, the client should exhibit symptoms within six months of the accident. 4. To meet the DSM-5 criteria for adjustment disorder, the client should exhibit symptoms within nine months of the accident.

2

A highly emotional client presents at an outpatient clinic appointment and states, "My dead husband returned to me during a séance." Which personality disorder should a nurse associate with this behavior? 1. Obsessive-compulsive personality disorder 2. Schizotypal personality disorder 3. Narcissistic personality disorder 4. Borderline personality disorder

2

A nurse is caring for a client who is experiencing a flat affect, paranoid delusions, anhedonia, anergia, neologisms, and echolalia. Which statement correctly differentiates the client's positive and negative symptoms of schizophrenia? 1. Paranoid delusions, anhedonia, and anergia are positive symptoms of schizophrenia. 2. Paranoid delusions, neologisms, and echolalia are positive symptoms of schizophrenia. 3. Paranoid delusions, anergia, and echolalia are negative symptoms of schizophrenia. 4. Paranoid delusions, flat affect, and anhedonia are negative symptoms of schizophrenia

2

A nursing instructor is teaching about the impaired nurse and the consequences of this impairment. Which statement by a student indicates that further instruction is needed? 1. "The state board of nursing must be notified with factual documentation of impairment." 2. "All state boards of nursing have passed laws that, under any circumstances, do not allow impaired nurses to practice." 3. "Many state boards of nursing require an impaired nurse to successfully complete counseling treatment programs prior to a return to work." 4. "After a return to practice, a recovering nurse may be closely monitored for several years."

2

A paranoid client presents with bizarre behaviors, neologisms, and thought insertion. Which nursing action should be prioritized to maintain this client's safety? 1. Assess for medication nonadherance. 2. Note escalating behaviors and intervene immediately. 3. Interpret attempts at communication. 4. Assess triggers for bizarre, inappropriate behaviors

2

After a teaching session about grief, a client says to the nurse, "I seem to be stuck in the anger stage of grieving over the loss of my son." How would the nurse assess this statement, and in what phase of the nursing process would this occur? 1. Assessment phase; nursing actions have been successful in achieving the objectives of care. 2. Evaluation phase; nursing actions have been successful in achieving the objectives of care. 3. Implementation phase; nursing actions have been successful in achieving the objectives of care. 4. Diagnosis phase; nursing actions have been successful in achieving the objectives of care

2

During a hiring interview, which response by a nursing applicant should indicate that the applicant operates from an ethical egoism framework? 1. "I would want to be treated in a caring manner if I were mentally ill." 2. "This job will pay the bills, and the workload is light enough for me." 3. "I will be happy caring for the mentally ill. Working in med/surg kills my back." 4. "It is my duty in life to be a psychiatric nurse. It is the right thing to do."

2

Group therapy is strongly encouraged, but not mandatory, in an inpatient psychiatric unit. The unit manager's policy is that clients can make a choice about whether or not to attend group therapy. Which ethical principle does the unit manager's policy preserve? 1. Justice 2. Autonomy 3. Veracity 4. Beneficence

2

Which is the basic premise of a recovery model used to treat clients diagnosed with bipolar disorder? 1. Medication adherence 2. Empowerment of the consumer 3. Total absence of symptoms 4. Improved psychosocial relationships

2

Which potential client should a nurse identify as a candidate for involuntarily commitment? 1. The client living under a bridge in a cardboard box. 2. The client threatening to commit suicide. 3. The client who never bathes and wears a wool hat in the summer. 4. The client who eats waste out of a garbage can.

2

Which situation exemplifies both assault and battery? 1. The nurse becomes angry, calls the client offensive names, and withholds treatment. 2. The nurse threatens to "tie down" the client and then does so, against the client's wishes. 3. The nurse hides the client's clothes and medicates the client to prevent elopement. 4. The nurse restrains the client without just cause and communicates this to family.

2

Which situation is an example of selective amnesia? 1. A client cannot relate any lifetime memories. 2. A client can describe driving to Ohio but cannot remember the car accident that occurred. 3. A client often wanders aimlessly after sunset. 4. A client cannot provide personal demographic information during admission assessment.

2

Which would be considered an appropriate outcome when planning care for an inpatient client diagnosed with SSD? 1. The client will admit to fabricating physical symptoms to gain benefits by day three. 2. The client will list three potential adaptive coping strategies to deal with stress by day two. 3. The client will comply with medical treatments for physical symptoms by day three. 4. The client will openly discuss physical symptoms with staff by day four.

2

After reporting a sexual assault, a female soldier is diagnosed with a personality disorder. Which of the following consequences may result? (Select all that apply.) 1. Court-martial proceedings 2. Loss of health-care benefits 3. Loss of service-related disability compensation 4. Stigma of a psychiatric diagnosis 5. Service discharge

2, 3, 4, 5

Which of the following instructions regarding lithium therapy should be included in a nurse's discharge teaching? (Select all that apply.) 1. Restrict sodium content. 2. Maintain a consistent sodium intake. 3. Avoid excessive use of beverages containing caffeine. 4. Restrict fluids to 1,500 mL per day. 5. Consume at least 2,500 to 3,000 mL of fluid per day.

2,3,5

A client began taking lithium carbonate (Lithobid) for the treatment of bipolar disorder approximately 1 month ago. The client asks if it is normal to have gained 12 pounds in this time frame. Which is the appropriate nursing response? 1. "That's strange. Weight loss is the typical pattern." 2. "What have you been eating? Weight gain is not usually associated with lithium." 3. "Weight gain is a common, but troubling, side effect." 4. "Weight gain only occurs during the first month of treatment with this drug."

3

A client diagnosed with DID switches personalities when confronted with destructive behavior. The nurse recognizes that this dissociation serves which function? 1. It is a means to attain secondary gain. 2. It is a means to explore feelings of excessive and inappropriate guilt. 3. It serves to isolate painful events so that the primary self is protected. 4. It serves to establish personality boundaries and limit inappropriate impulses.

3

A client diagnosed with a gambling disorder asks the nurse about medications that may be ordered by the client's physician to treat this disorder. The nurse would give the client information on which medications? 1. Escitalopram (Lexapro) and clozapine (Clozaril) 2. Citalopram (Celexa) and olanzapine (Zyprexa) 3. Lithium carbonate (Lithobid) and sertraline (Zoloft) 4. Naltrexone (ReVia) and ziprasidone (Geodon)

3

A client diagnosed with schizophrenia refuses to take medication, citing the right of autonomy. Under which circumstance would a nurse have the right to medicate the client against the client's wishes? 1. A client makes inappropriate sexual innuendos to a staff member. 2. A client constantly demands attention from the nurse by begging, "Help me get better." 3. A client physically attacks another client after being confronted in group therapy. 4. A client refuses to bathe or perform hygienic activities.

3

A client diagnosed with somatic symptom disorder (SSD) is most likely to exhibit which personality disorder characteristics? 1. Experiences intense and chaotic relationships with fluctuating attitudes toward others. 2. Socially irresponsible, exploitative, guiltless, and disregards rights of others. 3. Self-dramatizing, attention seeking, overly gregarious, and seductive. 4. Uncomfortable in social situations, perceived as timid, withdrawn, cold, and strange.

3

A client is diagnosed with schizophrenia spectrum disorder. A physician orders haloperidol (Haldol) 50 mg bid, benztropine (Cogentin) 1 mg prn, and zolpidem (Ambien) 10 mg HS. Which client behavior would warrant the nurse to administer benztropine? 1. Tactile hallucinations 2. Tardive dyskinesia 3. Restlessness and muscle rigidity 4. Reports of hearing disturbing voices

3

A client who will be receiving electroconvulsive therapy (ECT) must provide informed consent. Which situation should cause a nurse to question the validity of the informed consent? 1. The client is paranoid. 2. The client is 87 years old. 3. The client incorrectly reports his or her spouse's name, date, and time of day. 4. The client relies on his or her spouse to interpret the information.

3

A client's wife has been making excuses for her alcoholic husband's work absences. In family therapy, she states, "His problems at work are my fault." Which is the appropriate nursing response? 1. "Why do you assume responsibility for his behaviors?" 2. "I think you should start to confront his behavior." 3. "Your husband needs to deal with the consequences of his drinking." 4. "Do you understand what the term enabler means?"

3

A nurse is assessing a client diagnosed with substance induced psychotic disorder (SIPD). What would differentiate this client's symptoms from the symptoms of a client diagnosed with brief psychotic disorder (BPD)? 1. Clients diagnosed with SIPD experience delusions, whereas clients diagnosed with BPD do not. 2. Clients diagnosed with BPD experience hallucinations, whereas clients diagnosed with SIPD do not. 3. Catatonic features may be associated with SIPD, whereas BPD has no catatonic features. 4. Catatonic features may be associated with BPD, whereas SIPD has no catatonic features

3

A nursing instructor is teaching about bipolar disorders. Which statement differentiates the DSM-5 diagnostic criteria of a manic episode from a hypomanic episode? 1. During a manic episode, clients may experience an inflated self-esteem or grandiosity, and these symptoms are absent in hypomania. 2. During a manic episode, clients may experience a decreased need for sleep, and this symptom is absent in hypomania. 3. During a manic episode, clients may experience psychosis, and this symptom is absent in hypomania. 4. During a manic episode, clients may experience flight of ideas and racing thoughts, and these symptoms are absent in hypomania

3

A nursing instructor is teaching about suicide among active duty military. Which fact should the instructor include in the lesson plan? 1. On average, two suicides a day occur in the U.S. military. 2. From 2005 to 2009, relationship distress factored in more than 25% of Army suicides. 3. Statistically, in 2012, suicide rates of service members surpassed the number killed in combat. 4. Military suicides are associated with a narcissistic personality disorder diagnosis.

3

A nursing instructor is teaching about the Roberts' Seven-Stage Crisis Intervention Model. Which nursing action should be identified with Stage IV? 1. Collaboratively implement an action plan. 2. Help the client identify the major problems or crisis precipitants. 3. Help the client deal with feelings and emotions. 4. Collaboratively generate and explore alternatives

3

A paranoid client diagnosed with schizophrenia spectrum disorder states, "My psychiatrist is out to get me. I'm sad that the voice is telling me to stop him." What symptom is the client exhibiting, and what is the nurse's legal responsibility related to this symptom? 1. Magical thinking; administer an antipsychotic medication. 2. Persecutory delusions; orient the client to reality. 3. Command hallucinations; warn the psychiatrist. 4. Altered thought processes; call an emergency treatment team

3

An aging client diagnosed with schizophrenia spectrum disorder takes an antipsychotic and a beta-adrenergic blocking agent for hypertension. Understanding the combined side effects of these drugs, which statement by a nurse is most appropriate? 1. "Make sure you concentrate on taking slow, deep, cleansing breaths." 2. "Watch your diet and try to engage in some regular physical activity." 3. "Rise slowly when you change position from lying to sitting or sitting to standing." 4. "Wear sunscreen and try to avoid midday sun exposure."

3

As the sole survivor of a roadside bombing, a veteran is experiencing extreme guilt. Which nursing diagnosis would address this client's symptom? 1. Anxiety 2. Altered thought processes 3. Complicated grieving 4. Altered sensory perception

3

Parents ask a nurse how they should reply when their child, diagnosed with schizophrenia spectrum disorder, tells them that voices command him to harm others. Which is the appropriate nursing response? 1. "Tell him to stop discussing the voices." 2. "Ignore what he is saying, while attempting to discover the underlying cause." 3. "Focus on the feelings generated by the hallucinations and present reality." 4. "Present objective evidence that the voices are not real."

3

Research has shown that an adolescent (13 to18 years) would typically exhibit which behavior as a reaction to parental military deployment? 1. May exhibit regressive behaviors and assume blame for parent's departure. 2. May become sullen, tearful, throw temper tantrums, or develop sleep problems. 3. May participate in high-risk behaviors, sexual acting out, and drug or alcohol abuse. 4. May respond to schedule disruptions with irritability and/or apathy and weight loss.

3

What is the expected feeling and/or behavior experienced by military families during the "sustainment" cycle of deployment, as described by Pincus and associates? 1. Feelings alternate between denial and anticipation of loss. 2. Feelings alternate between excitement and apprehension associated with homecoming. 3. Feelings focus on the establishment of new support systems and new family routines. 4. Feelings focus on the struggle to take charge of the details of the new family structure.

3

Which client symptoms should lead a nurse to suspect a diagnosis of obsessive-compulsive personality disorder? 1. The client experiences unwanted, intrusive, and persistent thoughts. 2. The client experiences unwanted, repetitive behavior patterns. 3. The client experiences inflexibility and lack of spontaneity when dealing with others. 4. The client experiences obsessive thoughts that are externally imposed.

3

Which client would a nurse recognize as being at highest risk for the development of an AD? 1. A young married woman 2. An elderly unmarried man 3. A young unmarried woman 4. A young unmarried man

3

Which factors differentiate a client diagnosed with social phobia from a client diagnosed with schizoid personality disorder? 1. Clients diagnosed with social phobia are treated with cognitive behavioral therapy, whereas clients diagnosed with schizoid personality disorder need medications. 2. Clients diagnosed with schizoid personality disorder experience anxiety only in social settings, whereas clients diagnosed with social phobia experience generalized anxiety. 3. Clients diagnosed with social phobia avoid attending birthday parties, whereas clients diagnosed with schizoid personality disorder would isolate self on a continual basis. 4. Clients diagnosed with schizoid personality disorder avoid attending birthday parties, whereas clients diagnosed with social phobia would isolate self on a continual basis.

3

A nurse is assessing an adolescent client diagnosed with cyclothymic disorder. Which of the following DSM-5 diagnostic criteria would the nurse expect this client to meet? (Select all that apply.) . Numerous periods with manic symptoms 2. Possible comorbid diagnosis of a delusional disorder 3. Symptoms cause clinically significant impairment in important areas of functioning 4. Symptoms lasting for a minimum of two years 5. Depressive symptoms that do not meet the criteria for major depressive episode

3,5

A 20-year-old client and a 60-year-old client have had drunk driving accidents and are both experiencing extreme anxiety. From a psychosocial theory perspective, which of these clients would be predisposed to the diagnosis of adjustment disorder? 1. The 60-year-old, because of memory deficits. 2. The 60-year-old, because of decreased cognitive processing ability. 3. The 20-year-old, because of limited cognitive experiences. 4. The 20-year-old, because of lack of developmental maturity.

4

A client diagnosed with bipolar I disorder is exhibiting severe manic behaviors. A physician prescribes lithium carbonate (Eskalith) and olanzapine (Zyprexa). The client's spouse questions the Zyprexa order. Which is the appropriate nursing response? 1. "Zyprexa in combination with Eskalith cures manic symptoms." 2. "Zyprexa prevents extrapyramidal side effects." 3. "Zyprexa increases the effectiveness of the immune system." 4. "Zyprexa calms hyperactivity until the Eskalith takes effect."

4

A client is diagnosed with DID. What is the primary goal of therapy for this client? 1. To recover memories and improve thinking patterns. 2. To prevent social isolation. 3. To decrease anxiety and need for secondary gain. 4. To collaborate among sub-personalities to improve functioning.

4

A nurse is assessing a client diagnosed with schizophrenia spectrum disorder. The nurse asks the client, "Do you receive special messages from certain sources, such as the television or radio?" The nurse is assessing which potential symptom of this disorder? 1. Thought insertion 2. Paranoid delusions 3. Magical thinking 4. Delusions of reference

4

A nursing instructor is discussing various challenges in the treatment of clients diagnosed with bipolar disorder. Which student statement demonstrates an understanding of the most critical challenge in the care of these clients? 1. "Treatment is compromised when clients can't sleep." 2. "Treatment is compromised when irritability interferes with social interactions." 3. "Treatment is compromised when clients have no insight into their problems." 4. "Treatment is compromised when clients choose not to take their medications." 4. "Treatment is compromised when clients choose not to take their medications."

4

A nursing instructor is teaching about the DSM-5 diagnosis of depersonalization-derealization disorder (D-DD). Which student statement indicates a need for further instruction? 1. "Clients with this disorder can experience emotional and/or physical numbing and a distorted sense of time." 2. "Clients with this disorder can experience unreality or detachment with respect to their surroundings." 3. "During the course of this disorder, individuals or objects are experienced as dreamlike, foggy, lifeless, or visually distorted." 4. "During the course of this disorder, the client is out of touch with reality and is impaired in social, occupational, or other areas of functioning."

4

A nursing instructor is teaching about the etiology of IAD from a psychoanalytical perspective. What student statement about clients diagnosed with this disorder indicates that learning has occurred? 1. "They tend to have a familial predisposition to this disorder." 2. "When the sick role relieves them from stressful situations, their physical symptoms are reinforced." 3. "They misinterpret and cognitively distort their physical symptoms." 4. "They express personal worthlessness through physical symptoms, because physical problems are more acceptable than psychological problems."

4

A nursing instructor is teaching about trauma and stressor-related disorders. Which student statement indicates that further instruction is needed? 1. "The trauma that women experience is more likely to be sexual assault and child sexual abuse." 2. "The trauma that men experience is more likely to be accidents, physical assaults, combat, or viewing death or injury." 3. "After exposure to a traumatic event, only 10 percent of victims develop post-traumatic stress disorder (PTSD)." 4. "Research shows that PTSD is more common in men than in women."

4

An inpatient psychiatric physician refuses to treat clients without insurance and prematurely discharges those whose insurance benefits have expired. Which ethical principle should a nurse determine has been violated based on these actions? 1. Autonomy 2. Beneficence 3. Nonmaleficence 4. Justice

4

During an interview, which client statement should indicate to a nurse a potential diagnosis of schizotypal personality disorder? 1. "I don't have a problem. My family is inflexible, and relatives are out to get me." 2. "I am so excited about working with you. Have you noticed my new nail polish, 'Ruby Red Roses'?" 3. "I spend all my time tending my bees. I know a whole lot of information about bees." 4. "I am getting a message from the beyond that we have been involved with each other in a previous life.

4

Which are examples of primary and secondary gains that clients diagnosed with SSD: predominately pain, may experience? 1. Primary: chooses to seek a new doctor; Secondary: euphoric feeling from new medications 2. Primary: euphoric feeling from new medications; Secondary: chooses to seek a new doctor 3. Primary: receives get-well cards; Secondary: pain prevents attending stressful family reunion 4. Primary: pain prevents attending stressful family reunion; Secondary: receives get-well cards

4

Which client statement demonstrates positive progress toward recovery from a substance use disorder? 1. "I have completed detox and therefore am in control of my drug use." 2. "I will faithfully attend Narcotic Anonymous (NA) when I can't control my cravings." 3. "As a church deacon, my focus will now be on spiritual renewal." 4. "Taking those pills got out of control. It cost me my job, marriage, and children."

4

Which clinical presentation is associated with the most commonly diagnosed adjustment disorder (AD)? 1. Anxiety, feelings of hopelessness, and worry 2. Truancy, vandalism, and fighting 3. Nervousness, worry, and jitteriness 4. Depressed mood, tearfulness, and hopelessness

4

Which nursing diagnosis should be prioritized when providing nursing care to a client diagnosed with avoidant personality disorder? 1. Risk for violence: directed toward others R/T paranoid thinking 2. Risk for suicide R/T altered thought 3. Altered sensory perception R/T increased levels of anxiety 4. Social isolation R/T inability to relate to others

4

Which statement should a nurse identify as correct regarding a client's right to refuse treatment? 1. Clients can refuse pharmacological but not psychological treatment. 2. Clients can refuse any treatment at any time. 3. Clients can refuse only electroconvulsive therapy (ECT). 4. Professionals can override treatment refusal by an actively suicidal or homicidal client.

4

Which of the following rationales by a nurse explain to parents why is it difficult to diagnose a child or adolescent exhibiting symptoms of bipolar disorder? (Select all that apply.) 1. The diagnosis of bipolar disorder cannot be assigned prior to the age of 18. 2. Genetic predisposition is not a reliable diagnostic determinant. 3. Neurotransmitter levels vary considerably in accordance with age. 4. Children are naturally active, energetic, and spontaneous. 5. Bipolar symptoms mimic attention deficit-hyperactivity disorder symptoms.

4,5

. A nurse in the CCU is caring for a patient with HF who has developed an intracardiac thrombus. This creates a high risk for what sequela? A) Stroke B) Myocardial infarction (MI) C) Hemorrhage D) Peripheral edema

A

A 25-year-old man is involved in a motorcycle accident and injures his arm. The physician diagnoses the man with an intra-articular fracture and splints the injury. The nurse implements the teaching plan developed for this patient. What sequela of intra-articular fractures should the nurse describe regarding this patient? A) Post-traumatic arthritis B) Fat embolism syndrome (FES) C) Osteomyelitis D) Compartment syndrome

A

A 32-year-old patient comes to the clinic complaining of shoulder tenderness, pain, and limited movement. Upon assessment the nurse finds edema. An MRI shows hemorrhage of the rotator cuff tendons and the patient is diagnosed with impingement syndrome. What action should the nurse recommend in order to promote healing? A) Support the affected arm on pillows at night. B) Take prescribed corticosteroids as ordered. C) Put the shoulder through its full range of motion 3 times daily. D) Keep the affected arm in a sling for 2 to 4 weeks.

A

A client diagnosed with AD has been assigned the nursing diagnosis of anxiety R/T divorce. Which correctly written outcome addresses this client's problem? A. Rates anxiety as 4 out of 10 by discharge. B. States anxiety level has decreased by day one. C. Accomplishes activities of daily living independently. D. Demonstrates ability for adequate social functioning by day three.

A

A client diagnosed with chronic migraine headaches is considering acupuncture. The client asks a clinic nurse, "How does this treatment work?" Which is the best response by the nurse? A. "Western medicine believes that acupuncture stimulates the body's release of pain-fighting chemicals called endorphins." B. "I'm not sure why he suggested acupuncture. There are a lot of risks, including HIV." C. "Acupuncture works by encouraging the body to increase its development of serotonin and norepinephrine." D. "Your acupuncturist is your best resource for answering your specific questions."

A

A client diagnosed with depression and substance use disorder has an altered sleep pattern and demands that a psychiatrist prescribe a sedative. Which rationale explains why a nurse should encourage the client to first try nonpharmacological interventions? A. Sedative-hypnotics are potentially addictive and will lose their effectiveness due to tolerance. B. Sedative-hypnotics are expensive and have numerous side effects. C. Sedative-hypnotics interfere with necessary REM (rapid eye movement) sleep. D. Sedative-hypnotics are not as effective to promote sleep as antidepressant medications.

A

A football player is thought to have sustained an injury to his kidneys from being tackled from behind. The ER nurse caring for the patient reviews the initial orders written by the physician and notes that an order to collect all voided urine and send it to the laboratory for analysis. The nurse understands that this nursing intervention is important for what reason? A) Hematuria is the most common manifestation of renal trauma and blood losses may be microscopic, so laboratory analysis is essential. B) Intake and output calculations are essential and the laboratory will calculate the precise urine output produced by this patient. C) A creatinine clearance study may be ordered at a later time and the laboratory will hold all urine until it is determined if the test will be necessary. D) There is great concern about electrolyte imbalances and the laboratory will monitor the urine for changes in potassium and sodium concentrations.

A

A group of nurses have attended an inservice on the prevention of occupationally acquired diseases that affect healthcare providers. What action has the greatest potential to reduce a nurse's risk of acquiring hepatitis C in the workplace? A. Disposing of sharps appropriately and not recapping needles B. Performing meticulous hand hygiene at the appropriate moments in care C. Adhering to the recommended schedule of immunizations D. Wearing an N95 mask when providing care for patients on airborne precautions

A

A hospital's emergency operations plan has been enacted following an industrial accident. While one nurse performs the initial triage, what should other emergency medical services personnel do? A) Perform life-saving measures. B) Classify patients according to acuity. C) Provide health promotion education. D) Modify the emergency operations plan.

A

A lonely, depressed divorce has been self-medicating with cocaine for the past year. Which term should a nurse use to best describe this individuals situation? A. The individual is experiencing psychological addiction. B. The individual is experiencing physical addiction. C. The individual is experiencing substance addiction. D. The individual is experiencing social addiction.

A

A major earthquake has occurred within the vicinity of the local hospital. The nursing supervisor working the night shift at the hospital receives information that the hospital disaster plan will be activated. The supervisor will need to work with what organization responsible for coordinating interagency relief assistance? A) Office of Emergency Management B) Incident Command System C) Centers for Disease Control and Prevention (CDC) D) American Red Cross

A

A nurse has reported for a shift at a busy burns and plastics unit in a large university hospital. Which patient is most likely to have life-threatening complications? A) A 4-year-old scald victim burned over 24% of the body B) A 27-year-old male burned over 36% of his body in a car accident C) A 39-year-old female patient burned over 18% of her body D) A 60-year-old male burned over 16% of his body in a brush fire

A

A nurse is assessing a patient with an acoustic neuroma who has been recently admitted to an oncology unit. What symptoms is the nurse likely to find during the initial assessment? A) Loss of hearing, tinnitus, and vertigo B) Loss of vision, change in mental status, and hyperthermia C) Loss of hearing, increased sodium retention, and hypertension D) Loss of vision, headache, and tachycardia

A

A nurse is caring for a patient in the emergent/resuscitative phase of burn injury. During this phase, the nurse should monitor for evidence of what alteration in laboratory values? A) Sodium deficit B) Decreased prothrombin time (PT) C) Potassium deficit D) Decreased hematocrit

A

A nurse is caring for a patient who has a diagnosis of acute leukemia. What assessment most directly addresses the most common cause of death among patients with leukemia? A) Monitoring for infection B) Monitoring nutritional status C) Monitor electrolyte levels D) Monitoring liver function

A

A nurse is caring for a patient who is 12 hours postoperative following foot surgery. The nurse assesses the presence of edema in the foot. What nursing measure will the nurse implement to control the edema? A) Elevate the foot on several pillows. B) Apply warm compresses intermittently to the surgical area. C) Administer a loop diuretic as ordered. D) Increase circulation through frequent ambulation

A

A nurse is caring for a patient who is 12 hours postoperative following foot surgery. The nurse assesses the presence of edema in the foot. What nursing measure will the nurse implement to control the edema? A) Elevate the foot on several pillows. B) Apply warm compresses intermittently to the surgical area. C) Administer a loop diuretic as ordered. D) Increase circulation through frequent ambulation.

A

A nurse is caring for a patient who is exhibiting ventricular tachycardia (VT). Because the patient is pulseless, the nurse should prepare for what intervention? A) Defibrillation B) ECG monitoring C) Implantation of a cardioverter defibrillator D) Angioplasty

A

A nurse is caring for a patient with Hodgkin lymphoma at the oncology clinic. The nurse should be aware of what main goal of care? A) Cure of the disease B) Enhancing quality of life C) Controlling symptoms D) Palliation

A

A nurse is caring for a patient with hepatic encephalopathy. While making the initial shift assessment, the nurse notes that the patient has a flapping tremor of the hands. The nurse should document the presence of what sign of liver disease? A. Asterixis B. Constructional apraxia C. Fector hepaticus D. Palmar erythema

A

A nurse is caring for an adult patient diagnosed with a back strain. What health education should the nurse provide to this patient? A) Avoid lifting more than one-third of body weight without assistance. B) Focus on using back muscles efficiently when lifting heavy objects. C) Lift objects while holding the object a safe distance from the body. D) Tighten the abdominal muscles and lock the knees when lifting of an object.

A

A nurse is writing a care plan for a patient admitted to the emergency department (ED) with an open fracture. The nurse will assign priority to what nursing diagnosis for a patient with an open fracture of the radius? A) Risk for Infection B) Risk for Ineffective Role Performance C) Risk for Perioperative Positioning Injury D) Risk for Powerlessness

A

A nurse would recognize which treatment as most commonly used for AD and its appropriate rationale? 1. Psychotherapy; to examine the stressor and confront unresolved issues 2. Fluoxetine (Prozac); to stabilize mood and resolve symptoms 3. Eye movement desensitization therapy; to reprocess traumatic events 4. Lorazepam (Ativan); a first-line treatment to address symptoms of anxiety

A

A nursing student having no knowledge of alternative treatments states, "Aren't these therapies 'bogus' and, like a fad, will eventually fade away?" Which is an accurate nursing reply? A. "Like nursing, complementary therapies take a holistic approach to healing." B. "The American Nurses Association is researching the effectiveness of these therapies." C. "It is important to remain nonjudgmental about these therapies." D. "Alternative therapy concepts are rooted in psychoanalysis."

A

A patient admitted to the medical unit with HF is exhibiting signs and symptoms of pulmonary edema. The nurse is aware that positioning will promote circulation. How should the nurse best position the patient? A) In a high Fowlers position B) On the left side-lying position C) In a flat, supine position D) In the Trendelenburg position

A

A patient has a recent diagnosis of chronic pancreatitis and is undergoing diagnostic testing to determine pancreatic islet cell function. The nurse should anticipate what diagnostic test? A) Glucose tolerance test B) ERCP C) Pancreatic biopsy D) Abdominal ultrasonography

A

A patient has been admitted to a burn intensive care unit with extensive full-thickness burns over 25% of the body. After ensuring cardiopulmonary stability, what would be the nurses immediate, priority concern when planning this patients care? A) Fluid status B) Risk of infection C) Nutritional status D) Psychosocial coping

A

A patient has been brought to the ED with multiple trauma after a motor vehicle accident. After immediate threats to life have been addressed, the nurse and trauma team should take what action? A) Perform a rapid physical assessment. B) Initiate health education. C) Perform diagnostic imaging. D) Establish the circumstances of the accident.

A

A patient has been exposed to a nerve agent in a biochemical terrorist attack. This type of agent bonds with acetylcholinesterase, so that acetylcholine is not inactivated. What is the pathologic effect of this type of agent? A) Hyperstimulation of the nerve endings B) Temporary deactivation of the nerve endings C) Binding of the nerve endings D) Destruction of the nerve endings

A

A patient has been living with dilated cardiomyopathy for several years but has experienced worsening symptoms despite aggressive medical management. The nurse should anticipate what potential treatment? A) Heart transplantation B) Balloon valvuloplasty C) Cardiac catheterization D) Stent placement

A

A patient has questioned the nurses administration of IV normal saline, asking whether sterile water would be a more appropriate choice than saltwater. Under what circumstances would the nurse administer electrolyte-free water intravenously? A) Never, because it rapidly enters red blood cells, causing them to rupture. B) When the patient is severely dehydrated resulting in neurologic signs and symptoms C) When the patient is in excess of calcium and/or magnesium ions D) When a patients fluid volume deficit is due to acute or chronic renal failure

A

A patient has returned to the postsurgical unit from the PACU after an above-the-knee amputation of the right leg. Results of the nurse's initial postsurgical assessment were unremarkable but the patient has called out. The nurse enters the room and observes copious quantities of blood at the surgical site. What should be the nurse's initial action? A) Apply a tourniquet. B) Elevate the residual limb. C) Apply sterile gauze. D) Call the surgeon.

A

A patient in the emergent/resuscitative phase of a burn injury has had blood work and arterial blood gases drawn. Upon analysis of the patients laboratory studies, the nurse will expect the results to indicate what? A) Hyperkalemia, hyponatremia, elevated hematocrit, and metabolic acidosis B) Hypokalemia, hypernatremia, decreased hematocrit, and metabolic acidosis C) Hyperkalemia, hypernatremia, decreased hematocrit, and metabolic alkalosis D) Hypokalemia, hyponatremia, elevated hematocrit, and metabolic alkalosis

A

A patient is brought by ambulance to the ED after suffering what the family thinks is a stroke. The nurse caring for this patient is aware that an absolute contraindication for thrombolytic therapy is what? A) Evidence of hemorrhagic stroke B) Blood pressure of 180/110 mm Hg C) Evidence of stroke evolution D) Previous thrombolytic therapy within the past 12 months

A

A patient is brought to the ED by two police officers. The patient was found unconscious on the sidewalk, with his face and hands covered in blood. At present, the patient is verbally abusive and is fighting the staff in the ED, but appears medically stable. The decision is made to place the patient in restraints. What action should the nurse perform when the patient is restrained? A) Frequently assess the patient's skin integrity. B) Inform the patient that he is likely to be charged with assault. C) Avoid interacting with the patient until the restraints are removed. D) Take the opportunity to perform a full physical assessment

A

A patient is brought to the emergency department with a burn injury. The nurse knows that the first systemic event after a major burn injury is what? A) Hemodynamic instability B) Gastrointestinal hypermotility C) Respiratory arrest D) Hypokalemia

A

A patient is postoperative day 1 following intracranial surgery. The nurses assessment reveals that the patients LOC is slightly decreased compared with the day of surgery. What is the nurses best response to this assessment finding? A) Recognize that this may represent the peak of post-surgical cerebral edema. B) Alert the surgeon to the possibility of an intracranial hemorrhage. C) Understand that the surgery may have been unsuccessful. D) Recognize the need to refer the patient to the palliative care team.

A

A patient presents to a clinic complaining of a leg ulcer that isn't healing; subsequent diagnostic testing suggests osteomyelitis. The nurse is aware that the most common pathogen to cause osteomyelitis is what? A) Staphylococcus aureus B) Proteus C) Pseudomonas D) Escherichia coli

A

A patient presents to the ED in distress and complaining of crushing chest pain. What is the nurses priority for assessment? A) Prompt initiation of an ECG B) Auscultation of the patients point of maximal impulse (PMI) C) Rapid assessment of the patients peripheral pulses D) Palpation of the patients cardiac apex

A

A patient tells the nurse that he has pain and numbness to his thumb, first finger, and second finger of the right hand. The nurse discovers that the patient is employed as an auto mechanic, and that the pain is increased while working. This may indicate that the patient could possibly have what health problem? A) Carpel tunnel syndrome B) Tendonitis C) Impingement syndrome D) Dupuytren's contracture

A

A patient the nurse is caring for has a permanent pacemaker implanted with the identification code beginning with VVI. What does this indicate? A) Ventricular paced, ventricular sensed, inhibited B) Variable paced, ventricular sensed, inhibited C) Ventricular sensed, ventricular situated, implanted D) Variable sensed, variable paced, inhibited

A

A patient who has undergone a lower limb amputation is preparing to be discharged home. What outcome is necessary prior to discharge? A) Patient can demonstrate safe use of assistive devices. B) Patient has a healed, nontender, nonadherent scar. C) Patient can perform activities of daily living independently. D) Patientis free of pain.

A

A patient who is being treated for pneumonia starts complaining of sudden shortness of breath. An arterial blood gas (ABG) is drawn. The ABG has the following values: pH 7.21, PaCO2 64 mm Hg, HCO3 = 24 mm Hg. What does the ABG reflect? A) Respiratory acidosis B) Metabolic alkalosis C) Respiratory alkalosis D) Metabolic acidosis

A

A patient who is in the acute phase of recovery from a burn injury has yet to experience adequate pain control. What pain management strategy is most likely to meet this patients needs? A) A patient-controlled analgesia (PCA) system B) Oral opioids supplemented by NSAIDs C) Distraction and relaxation techniques supplemented by NSAIDs D) A combination of benzodiazepines and topical anesthetics

A

A patient with HF has met with his primary care provider and begun treatment with an angiotensin-converting enzyme (ACE) inhibitor. When the patient begins treatment, the nurse should prioritize what assessment? A) Blood pressure B) Level of consciousness (LOC) C) Assessment for nausea D) Oxygen saturation

A

A patient with a C5 spinal cord injury is tetraplegic. After being moved out of the ICU, the patient complains of a severe throbbing headache. What should the nurse do first? A) Check the patient's indwelling urinary catheter for kinks to ensure patency. B) Lower the HOB to improve perfusion. C) Administer analgesia. D) Reassure the patient that headaches are expected after spinal cord injuries

A

A patient with a recent diagnosis of ITP has asked the nurse why the care team has not chosen to administer platelets, stating, I have low platelets, so why not give me a transfusion of exactly what Im missing? How should the nurse best respond? A) Transfused platelets usually arent beneficial because theyre rapidly destroyed in the body. B) A platelet transfusion often blunts your bodys own production of platelets even further. C) Finding a matching donor for a platelet transfusion is exceedingly difficult. D) A very small percentage of the platelets in a transfusion are actually functional.

A

A patient's abdominal ultrasound indicates cholelithiasis. When the nurse is reviewing the patient's laboratory studies, what finding is most closely associated with this diagnosis? A) Increased bilirubin B) Decreased serum cholesterol C) Increased blood urea nitrogen (BUN) D) Decreased serum alkaline phosphatase level

A

A patients low prothrombin time (PT) was attributed to a vitamin K deficiency and the patients PT normalized after administration of vitamin K. When performing discharge education in an effort to prevent recurrence, what should the nurse emphasize? A) The need for adequate nutrition B) The need to avoid NSAIDs C) The need for constant access to factor concentrate D) The need for meticulous hygiene

A

A psychiatric nurse working on an inpatient unit receives a call asking if an individual has been a client in the facility. Which nursing response reflects appropriate legal and ethical obligations? 1. The nurse refuses to give any information to the caller, citing rules of confidentiality. 2. The nurse hangs up on the caller. 3. The nurse confirms that the person has been at the facility but adds no additional information. 4. The nurse suggests that the caller speak to the client's therapist.

A

A rehabilitation nurse is working with a patient who has had a below-the-knee amputation. The nurse knows the importance of the patient's active participation in self-care. In order to determine the patient's ability to be an active participant in self-care, the nurse should prioritize assessment of what variable? A) The patient's attitude B) The patient's learning style C) The patient's nutritional status D) The patient's presurgical level of function

A

After a major ischemic stroke, a possible complication is cerebral edema. Nursing care during the immediate recovery period from an ischemic stroke should include which of the following? A) Positioning to avoid hypoxia B) Maximizing PaCO2 C) Administering hypertonic IV solution D) Initiating early mobilization

A

An 80-year-old man in a long-term care facility has a chronic leg ulcer and states that the area has become increasingly painful in recent days. The nurse notes that the site is now swollen and warm to the touch. The patient should undergo diagnostic testing for what health problem? A) Osteomyelitis B) Osteoporosis C) Osteomalacia D) Septic arthritis

A

An 83-year-old patient is brought in by ambulance from a long-term care facility. The patient's symptoms are weakness, lethargy, incontinence, and a change in mental status. The nurse knows that emergencies in older adults may be more difficult to manage. Why would this be true? A) Older adults may have an altered response to treatment. B) Older adults are often reluctant to adhere to prescribed treatment. C) Older adults have difficulty giving a health history. D) Older adults often stigmatize their peers who use the ED.

A

An older adult patient has been diagnosed with aortic regurgitation. What change in blood flow should the nurse expect to see on this patients echocardiogram? A) Blood to flow back from the aorta to the left ventricle B) Obstruction of blood flow from the left ventricle C) Blood to flow back from the left atrium to the left ventricle D) Obstruction of blood from the left atrium to left ventricle

A

An oncology nurse is providing health education for a patient who has recently been diagnosed with leukemia. What should the nurse explain about commonalities between all of the different subtypes of leukemia? A) The different leukemias all involve unregulated proliferation of white blood cells. B) The different leukemias all have unregulated proliferation of red blood cells and decreased bone marrow function. C) The different leukemias all result in a decrease in the production of white blood cells. D) The different leukemias all involve the development of cancer in the lymphatic system.

A

During a health education session, a participant has asked about the hepatitis E virus. What prevention measure should the nurse recommend for preventing infection with this virus? A. Following proper hand-washing techniques B. Avoiding chemicals that are toxic to the liver C. Wearing a condom during sexual contact D. Limiting alcohol intake

A

Emergency department (ED) staff members have been trained to follow steps that will decrease the risk of secondary exposure to a chemical. When conducting decontamination, staff members should remove the patient's clothing and then perform what action? A) Rinse the patient with water. B) Wash the patient with a dilute bleach solution. C) Wash the patient chlorhexidine. D) Rinse the patient with hydrogen peroxide.

A

Palliative care is based on interdisciplinary collaboration. How does interdisciplinary collaboration differ from multidisciplinary practice? 1. It is based on communication and cooperation. 2. It is based on expertise and patient preference. 3. It is based on the participation of clinicians without a team leader. 4. It is based on clinicians of varied backgrounds integrating their separate plans of care.

A

Renal failure can have prerenal, renal, or postrenal causes. A patient with acute kidney injury is being assessed to determine where, physiologically, the cause is. If the cause is found to be prerenal, which condition most likely caused it? A) Heart failure B) Glomerulonephritis C) Ureterolithiasis D) Aminoglycoside toxicity

A

The OR nurse is setting up a water-seal chest drainage system for a patient who has just had a thoracotomy. The nurse knows that the amount of suction in the system is determined by the water level. At what suction level should the nurse set the system? A) 20 cm H2O B) 15 cm H2O C) 10 cm H2O D) 5 cm H2O

A

The announcement is made that the facility may return to normal functioning after a local disaster. In the emergency operations plan, what is this referred to as? A) Demobilization response B) Post-incident response C) Crisis diffusion D) Reversion

A

The critical care nurse is caring for a patient with a pulmonary artery pressure monitoring system. The nurse is aware that pulmonary artery pressure monitoring is used to assess left ventricular function. What is an additional function of pulmonary artery pressure monitoring systems? A) To assess the patients response to fluid and drug administration B) To obtain specimens for arterial blood gas measurements C) To dislodge pulmonary emboli D) To diagnose the etiology of chronic obstructive pulmonary disease

A

The home health nurse is performing a home visit for an oncology patient discharged 3 days ago after completing treatment for non-Hodgkin lymphoma. The nurse's assessment should include examination for the signs and symptoms of what complication? A) Tumor lysis syndrome (TLS) B) Syndrome of inappropriate antiduretic hormone (SIADH) C) Disseminated intravascular coagulation (DIC) D) Hypercalcemia

A

The nurse and the other members of the team are caring for a patient who converted to ventricular fibrillation (VF). The patient was defibrillated unsuccessfully and the patient remains in VF. According to national standards, the nurse should anticipate the administration of what medication? A) Epinephrine 1 mg IV push B) Lidocaine 100 mg IV push C) Amiodarone 300 mg IV push D) Sodium bicarbonate 1 amp IV push

A

The nurse in a pediatric ICU is caring for a child who is dying of sickle cell anemia. The child's mother has been unable to eat or sleep and can talk only about her impending loss and the guilt she feels about the child's pain and suffering. What intervention has the highest priority? A) Allowing the patient to express her feelings without judging her B) Helping the patient to understand the phases of the grieving process C) Reassuring the patient that the child's death is not her fault D) Arranging for genetic counseling to inform the patient of her chances of having another child with the disease

A

The nurse in the ED is caring for a patient recently admitted with a likely myocardial infarction. The nurse understands that the patients heart is pumping an inadequate supply of oxygen to the tissues. For what health problem should the nurse assess? A) Dysrhythmias B) Increase in blood pressure C) Increase in heart rate D) Decrease in oxygen demands

A

The nurse in the ICU is admitting a 57-year-old man with a diagnosis of possible septic shock. The nurses assessment reveals that the patient has a normal blood pressure, increased heart rate, decreased bowel sounds, and cold, clammy skin. The nurses analysis of these data should lead to what preliminary conclusion? A) The patient is in the compensatory stage of shock. B) The patient is in the progressive stage of shock. C) The patient will stabilize and be released by tomorrow. D) The patient is in the irreversible stage of shock.

A

The nurse is assessing a 73-year-old patient who was diagnosed with metastatic prostate cancer. The nurse notes that the patient is exhibiting signs of loss, grief, and intense sadness. Based on this assessment data, the nurse will document that the patient is most likely in what stage of death and dying? A) Depression B) Denial C) Anger D) Resignation

A

The nurse is caring for a patient admitted with a drug overdose. What is the nurse's priority responsibility in caring for this patient? A) Support the patient's respiratory and cardiovascular function. B) Provide for the safety of the patient. C) Enhance clearance of the offending agent. D) Ensure the safety of the staff.

A

The nurse is caring for a patient in the ICU whose condition is deteriorating. The nurse receives orders to initiate an infusion of dopamine. What would be the priority assessment and interventions specific to the administration of vasoactive medications? A) Frequent monitoring of vital signs, monitoring the central line site, and providing accurate drug titration B) Reviewing medications, performing a focused cardiovascular assessment, and providing patient education C) Reviewing the laboratory findings, monitoring urine output, and assessing for peripheral edema D) Routine monitoring of vital signs, monitoring the peripheral IV site, and providing early discharge instructions

A

The nurse is caring for a patient who has terminal lung cancer and is unconscious. Which assessment finding would most clearly indicate to the nurse that the patient's death is imminent? A) Mottling of the lower limbs B) Slow, steady pulse C) Bowel incontinence D) Increased swallowing

A

The nurse is caring for a patient who is ready to be weaned from the ventilator. In preparing to assist in the collaborative process of weaning the patient from a ventilator, the nurse is aware that the weaning of the patient will progress in what order? A) Removal from the ventilator, tube, and then oxygen B) Removal from oxygen, ventilator, and then tube C) Removal of the tube, oxygen, and then ventilator D) Removal from oxygen, tube, and then ventilator

A

The nurse is caring for a patient who is scheduled for a lobectomy for a diagnosis of lung cancer. While assisting with a subclavian vein central line insertion, the nurse notes the clients oxygen saturation rapidly dropping. The patient complains of shortness of breath and becomes tachypneic. The nurse suspects a pneumothorax has developed. Further assessment findings supporting the presence of a pneumothorax include what? A) Diminished or absent breath sounds on the affected side B) Paradoxical chest wall movement with respirations C) Sudden loss of consciousness D) Muffled heart sounds

A

The nurse is caring for a patient who is to begin receiving external radiation for a malignant tumor of the neck. While providing patient education, what potential adverse effects should the nurse discuss with the patient? A) Impaired nutritional status B) Cognitive changes C) Diarrhea D) Alopecia

A

The nurse is caring for a patient who sustained a moderate head injury following a bicycle accident. The nurses most recent assessment reveals that the patients respiratory effort has increased. What is the nurses most appropriate response? A) Inform the care team and assess for further signs of possible increased ICP. B) Administer bronchodilators as ordered and monitor the patients LOC. C) Increase the patients bed height and reassess in 30 minutes. D) Administer a bolus of normal saline as ordered.

A

The nurse is caring for a patient whose recent health history includes an altered LOC. What should be the nurses first action when assessing this patient? A) Assessing the patients verbal response B) Assessing the patients ability to follow complex commands C) Assessing the patients judgment D) Assessing the patients response to pain

A

The nurse is caring for a patient with severe left ventricular dysfunction who has been identified as being at risk for sudden cardiac death. What medical intervention can be performed that may extend the survival of the patient? A) Insertion of an implantable cardioverter defibrillator B) Insertion of an implantable pacemaker C) Administration of a calcium channel blocker D) Administration of a beta-blocker

A

The nurse is performing a physical assessment on a patient suspected of having HF. The presence of what sound would signal the possibility of impending HF? A) An S3 heart sound B) Pleural friction rub C) Faint breath sounds D) A heart murmur

A

The nurse is planning the care of a patient who has been recently diagnosed with a cerebellar tumor. Due to the location of this patient's tumor, the nurse should implement measures to prevent what complication? A) Falls B) Audio hallucinations C) Respiratory depression D) Labile BP

A

The nurse is preparing to administer a unit of platelets to an adult patient. When administering this blood product, which of the following actions should the nurse perform? A) Administer the platelets as rapidly as the patient can tolerate. B) Establish IV access as soon as the platelets arrive from the blood bank. C) Ensure that the patient has a patent central venous catheter. D) Aspirate 10 to 15 mL of blood from the patient's IV immediately following the transfusion

A

The nurse is providing health education to a patient who has a C6 spinal cord injury. The patient asks why autonomic dysreflexia is considered an emergency. What would be the nurse's best answer? A) "The sudden increase in BP can raise the ICP or rupture a cerebral blood vessel." B) "The suddenness of the onset of the syndrome tells us the body is struggling to maintain its normal state." C) "Autonomic dysreflexia causes permanent damage to delicate nerve fibers that are healing." D) "The sudden, severe headache increases muscle tone and can cause further nerve damage."

A

The nurse is reviewing the medication administration record of a patient diagnosed with systolic HF. What medication should the nurse anticipate administering to this patient? A) A beta-adrenergic blocker B) An antiplatelet aggregator C) A calcium channel blocker D) A nonsteroidal anti-inflammatory drug (NSAID)

A

The nurse, a member of the health care team in the ED, is caring for a patient who is determined to be in the irreversible stage of shock. What would be the most appropriate nursing intervention? A) Provide opportunities for the family to spend time with the patient, and help them to understand the irreversible stage of shock. B) Inform the patients family immediately that the patient will likely not survive to allow the family time to make plans and move forward. C) Closely monitor fluid replacement therapy, and inform the family that the patient will probably survive and return to normal life. D) Protect the patients airway, optimize intravascular volume, and initiate the early rehabilitation process.

A

The physician has placed a central venous pressure (CVP) monitoring line in an acutely ill patient so right ventricular function and venous blood return can be closely monitored. The results show decreased CVP. What does this indicate? A) Possible hypovolemia B) Possible myocardial infarction (MI) C) Left-sided heart failure D) Aortic valve regurgitation

A

What should be the priority nursing diagnosis for a client experiencing alcohol withdrawal? A. Risk for injury R/T central nervous system stimulation B. Disturbed thought processes R/T tactile hallucinations C. Ineffective coping R/T powerlessness over alcohol use D. Ineffective denial R/T continued alcohol use despite negative consequences

A

When preparing to discharge a patient home, the nurse has met with the family and warned them that the patient may exhibit unexpected emotional responses. The nurse should teach the family that these responses are typically a result of what cause? A) Frustration around changes in function and communication B) Unmet physiologic needs C) Changes in brain activity during sleep and wakefulness D) Temporary changes in metabolism

A

While a patient is receiving IV doxorubicin hydrochloride for the treatment of cancer, the nurse observes swelling and pain at the IV site. The nurse should prioritize what action? A) Stopping the administration of the drug immediately B) Notifying the patient's physician C) Continuing the infusion but decreasing the rate D) Applying a warm compress to the infusion site

A

You are caring for a 50-year-old man diagnosed with multiple myeloma; he has just been told by the care team that his prognosis is poor. He is tearful and trying to express his feelings, but he is having difficulty. What should you do first? A) Ask if he would like for you to sit with him while he collects his thoughts B) Tell him that you will leave for now but will be back shortly C) Offer to call pastoral care or a member of his chosen clergy D) Reassure him that you can understand how he is feeling

A

You are caring for a patient admitted with a diagnosis of acute kidney injury. When you review your patients most recent laboratory reports, you note that the patients magnesium levels are high. You should prioritize assessment for which of the following health problems? A) Diminished deep tendon reflexes B) Tachycardia C) Cool, clammy skin D) Acute flank pain

A

You are the nurse caring for a 77-year-old male patient who has been involved in a motor vehicle accident. You and your colleague note that the patients labs indicate minimally elevated serum creatinine levels, which your colleague dismisses. What can this increase in creatinine indicate in older adults? A) Substantially reduced renal function B) Acute kidney injury C) Decreased cardiac output D) Alterations in ratio of body fluids to muscle mass

A

The nurse is admitting a patient to the unit who is scheduled for removal of an intracranial mass. What diagnostic procedures might be included in this patients admission orders? Select all that apply. A) Transcranial Doppler flow study B) Cerebral angiography C) MRI D) Cranial radiography E) Electromyelography (EMG)

A, B, C

Which statements represent positive outcomes for clients diagnosed with narcissistic personality disorder? (Select all that apply.) A. The client will relate one empathetic statement toward another client in group by day 2. B. The client will identify one personal limitation by day 1. C. The client will acknowledge one strength that another client possesses by day 2. D. The client will list four personal strengths by day 3. E. The client will list two lifetime achievements by discharge.

A, B, C

A nurse on the renal unit is caring for a patient who will soon begin peritoneal dialysis. The family of the patient asks for education about the peritoneal dialysis catheter that has been placed in the patient's peritoneum. The nurse explains the three sections of the catheter and talks about the two cuffs on the dialysis catheter. What would the nurse explain about the cuffs? Select all that apply. A) The cuffs are made of Dacron polyester. B) The cuffs stabilize the catheter. C) The cuffs prevent the dialysate from leaking. D) The cuffs provide a barrier against microorganisms. E) The cuffs absorb dialysate

A, B, C, D

An adult patient has sought care for the treatment of headaches that have become increasingly severe and frequent over the past several months. Which of the following questions addresses potential etiological factors? Select all that apply? A) Are you exposed to any toxins or chemicals at work? B) How would you describe your ability to cope with stress? C) What medications are you currently taking? D) When was the last time you were hospitalized? E) Does anyone else in your family struggle with headaches?

A, B, C, E

A patient with ESKD is scheduled to begin hemodialysis. The nurse is working with the patient to adapt the patient's diet to maximize the therapeutic effect and minimize the risks of complications. The patient's diet should include which of the following modifications? Select all that apply. A) Decreased protein intake B) Decreased sodium intake C) Increased potassium intake D) Fluid restriction E) Vitamin D supplementation

A, B, D

A patient with an inoperable brain tumor has been told that he has a short life expectancy. On what aspects of assessment and care should the home health nurse focus? Select all that apply. A) Pain control B) Management of treatment complications C) Interpretation of diagnostic tests D) Assistance with self-care E) Administration of treatments

A, B, D, E

The nurse is creating a care plan for a patient diagnosed with HF. When addressing the problem of anxiety, what interventions should the nurse include in the care plan? Select all that apply. A) Facilitate the presence of friends and family whenever possible. B) Teach the patient about the harmful effects of anxiety on cardiac function. C) Provide supplemental oxygen, as needed. D) Provide validation of the patients expressions of anxiety. E) Administer benzodiazepines two to three times daily.

A, C, D

A nurse is caring for a patient who is being assessed following complaints of severe and persistent low back pain. The patient is scheduled for diagnostic testing in the morning. Which of the following are appropriate diagnostic tests for assessing low back pain? that apply. A) Computed tomography (CT) B) Angiography C) Magnetic resonance imaging (MRI) D) Ultrasound E) X-ray

A, C, D, E

The nurse caring for a patient in a persistent vegetative state is regularly assessing for potential complications. Complications of neurologic dysfunction for which the nurse should assess include which of the following? Select all that apply. A) Contractures B) Hemorrhage C) Pressure ulcers D) Venous thromboembolism E) Pneumonia

A, C, D, E

The nurse providing care for a patient post PTCA knows to monitor the patient closely. For what complications should the nurse monitor the patient? Select all that apply. A) Abrupt closure of the coronary artery B) Venous insufficiency C) Bleeding at the insertion site D) Retroperitoneal bleeding E) Arterial occlusion

A, C, D, E

A patient is in the acute phase of a burn injury. One of the nursing diagnoses in the plan of care is Ineffective Coping Related to Trauma of Burn Injury. What interventions appropriately address this diagnosis? Select all that apply. A) Promote truthful communication. B) Avoid asking the patient to make decisions. C) Teach the patient coping strategies. D) Administer benzodiazepines as ordered. E) Provide positive reinforcement.

A, C, E

Which of the following practices should a nurse describe to a client as being incorporated during yoga therapy? (Select all that apply.) A. Deep breathing B. Meridian therapy C. Balanced body postures D. Massage therapy E. Meditation

A, C, E

During a patient's recovery from stroke, the nurse should be aware of predictors of stroke outcome in order to help patients and families set realistic goals. What are the predictors of stroke outcome? Select all that apply. A) National Institutes of Health Stroke Scale (NIHSS) score B) Race C) LOC at time of admission D) Gender E) Age

A, C,E

The intensive care nurse is responsible for the care of a patient with shock. What cardiac signs or symptoms would suggest to the nurse that the patient may be experiencing acute organ dysfunction? Select all that apply. A) Drop in systolic blood pressure of 40 mm Hg from baselines B) Hypotension that responds to bolus fluid resuscitation C) Exaggerated response to vasoactive medications D) Serum lactate >4 mmol/L E) Mean arterial pressure (MAP) of 65 mm Hg

A, D, E

A patient with spinal cord injury is ready to be discharged home. A family member asks the nurse to review potential complications one more time. What are the potential complications that should be monitored for in this patient? Select all that apply. A) Orthostatic hypotension B) Autonomic dysreflexia C) DVT D) Salt-wasting syndrome E) Increased ICP

A,B,C

The patient has just returned to the floor after balloon valvuloplasty of the aortic valve and the nurse is planning appropriate assessments. The nurse should know that complications following this procedure include what? Select all that apply. A) Emboli B) Mitral valve damage C) Ventricular dysrhythmia D) Atrial-septal defect E) Plaque formation

A,B,C

. A nurse is admitting a client who has been diagnosed with PTSD. Which of the following symptoms might the nurse expect to assess? (Select all that apply.) A. Feelings of guilt that precipitate social isolation B. Aggressive behavior that affects job performance C. Relationship problems D. High levels of anxiety E. Escalating symptoms lasting less than one month

A,B,C,D

A nurse is providing care for a patient who has a recent diagnosis of Paget's disease. When planning this patient's nursing care, interventions should address what nursing diagnoses? Select all that apply. A) Impaired Physical Mobility B) Acute Pain C) Disturbed Auditory Sensory Perception D) Risk for Injury E) Risk for Unstable Blood Glucos

A,B,C,D

The nurse is caring for a patient status after a motor vehicle accident. The patient has developed AKI. What is the nurse's role in caring for this patient? Select all that apply. A) Providing emotional support for the family B) Monitoring for complications C) Participating in emergency treatment of fluid and electrolyte imbalances D) Providing nursing care for primary disorder (trauma) E) Directing nutritional interventions

A,B,C,D

A nurse is planning the care of an older adult patient who will soon be discharged home after treatment for a fractured hip. In an effort to prevent future fractures, the nurse should encourage which of the following? Select all that apply. A) Regular bone density testing B) A high-calcium diet C) Use of falls prevention precautions D) Use of corticosteroids as ordered E) Weight-bearing exercise

A,B,C,E

An older adult patient experienced a fall and required treatment for a fractured hip on the orthopedic unit. Which of the following are contributory factors to the incidence of falls and fractured hips among the older adult population? Select all that apply. A) Loss of visual acuity B) Adverse medication effects C) Slowed reflexes D) Hearing loss E) Muscle weakness

A,B,C,E

A nurse educator is reviewing the indications for chest drainage systems with a group of medical nurses. What indications should the nurses identify? Select all that apply. A) Post thoracotomy B) Spontaneous pneumothorax C) Need for postural drainage D) Chest trauma resulting in pneumothorax E) Pleurisy

A,B,D

A nurse who provides care on an acute medical unit has observed that physicians are frequently reluctant to refer patients to hospice care. What are contributing factors that are known to underlie this tendency? Select all that apply. A) Financial pressures on health care providers B) Patient reluctance to accept this type of care C) Strong association of hospice care with prolonging death D) Advances in curative treatment in late-stage illness E) Ease of making a terminal diagnosis

A,B,D

A patient's physician has ordered a liver panel in response to the patient's development of jaundice. When reviewing the results of this laboratory testing, the nurse should expect to review what blood tests? Select all that apply. A. Alanine aminotransferase (ALT) B. C-reactive protein (CRP) C. Gamma-glutamyl transferase (GGT) D. Aspartate aminotransferase (AST) E. B-type natriuretic peptide (BNP)

A,B,D

The nurse is assessing a patient with acute coronary syndrome (ACS). The nurse includes a careful history in the assessment, especially with regard to signs and symptoms. What signs and symptoms are suggestive of ACS? Select all that apply. A) Dyspnea B) Unusual fatigue C) Hypotension D) Syncope E) Peripheral cyanosis

A,B,D

When hemodynamic monitoring is ordered for a patient, a catheter is inserted into the appropriate blood vessel or heart chamber. When assessing a patient who has such a device in place, the nurse should check which of the following components? Select all that apply. A) A transducer B) A flush system C) A leveler D) A pressure bag E) An oscillator

A,B,D

A critical care nurse is caring for a patient with a hemodynamic monitoring system in place. For what complications should the nurse assess? Select all that apply. A) Pneumothorax B) Infection C) Atelectasis D) Bronchospasm E) Air embolism

A,B,E

The nurse is describing some of the major characteristics of cancer to a patient who has recently received a diagnosis of malignant melanoma. When differentiating between benign and malignant cancer cells, the nurse should explain differences in which of the following aspects? Select all that apply. A) Rate of growth B) Ability to cause death C) Size of cells D) Cell contents E) Ability to spread

A,B,E

A patient's assessment and diagnostic testing are suggestive of acute pancreatitis. When the nurse is performing the health interview, what assessment questions address likely etiologic factors? Select all that apply. A) "How many alcoholic drinks do you typically consume in a week?" B) "Have you ever been tested for diabetes?" C) "Have you ever been diagnosed with gallstones?" D) "Would you say that you eat a particularly high-fat diet?" E) "Does anyone in your family have cystic fibrosis?

A,C

The results of a patients most recent blood work and physical assessment are suggestive of immune thrombocytopenic purpura (ITP). This patient should undergo testing for which of the following potential causes? Select all that apply. A) Hepatitis B) Acute renal failure C) HIV D) Malignant melanoma E) Cholecystitis

A,C

You are caring for an adult patient who has developed a mild oral yeast infection following chemotherapy. What actions should you encourage the patient to perform? Select all that apply. A) Use a lip lubricant. B) Scrub the tongue with a firm-bristled toothbrush. C) Use dental floss every 24 hours. D) Rinse the mouth with normal saline. E) Eat spicy food to aid in eradicating the yeast.

A,C,D

An oncology nurse recognizes a patients risk for fluid imbalance while the patient is undergoing treatment for leukemia. What relevant assessments should the nurse include in the patients plan of care? Select all that apply. A) Monitoring the patients electrolyte levels B) Monitoring the patients hepatic function C) Measuring the patients weight on a daily basis D) Measuring and recording the patients intake and output E) Auscultating the patients lungs frequently

A,C,D,E

A patient admitted to the ED with severe diarrhea and vomiting is subsequently diagnosed with food poisoning. The nurse caring for this patient assesses for signs and symptoms of fluid and electrolyte imbalances. For what signs and symptoms would this nurse assess? Select all that apply. A) Dysrhythmias B) Hypothermia C) Hypotension D) Hyperglycemia E) Delirium

A,C,E

The nurse is planning the care of a patient with HF. The nurse should identify what overall goals of this patient's care? A) Improve functional status B) Prevent endocarditis. C) Extend survival. D) Limit physical activity. E) Relieve patient symptoms.

A,C,E

You are doing discharge teaching with a patient who has hypophosphatemia during his time in hospital. The patient has a diet ordered that is high in phosphate. What foods would you teach this patient to include in his diet? Select all that apply. A) Milk B) Beef C) Poultry D) Green vegetables E) Liver

A,C,E

A team of nurses are reviewing the similarities and differences between the different classifications of shock. Which subclassifications of circulatory shock should the nurses identify? Select all that apply. A) Anaphylactic B) Hypovolemic C) Cardiogenic D) Septic E) Neurogenic

A,D,E

The school nurse is giving a presentation on preventing spinal cord injuries (SCI). What should the nurse identify as prominent risk factors for SCI? Select all that apply. A) Young age B) Frequent travel C) African American race D) Male gender E) Alcohol or drug use

A,D,E

A 25-year-old female patient with brain metastases is considering her life expectancy after her most recent meeting with her oncologist. Based on the fact that the patient is not receiving treatment for her brain metastases, what is the nurse's most appropriate action? A) Promoting the patient's functional status and ADLs B) Ensuring that the patient receives adequate palliative care C) Ensuring that the family does not tell the patient that her condition is terminal D) Promoting adherence to the prescribed medication regimen

B

A 71-year-old patient with ESKD has been told by the physician that it is time to consider hemodialysis until a transplant can be found. The patient tells the nurse she is not sure she wants to undergo a kidney transplant. What would be an appropriate response for the nurse to make? A) "The decision is certainly yours to make, but be sure not to make a mistake." B) "Kidney transplants in patients your age are as successful as they are in younger patients." C) "I understand your hesitancy to commit to a transplant surgery. Success is comparatively rare." D) "Have you talked this over with your family

B

A client inquires about the practice of therapeutic touch. Which nursing reply best explains the goal of this therapy? A. "The goal is to improve circulation to the body by deep, circular massage." B. "The goal is to re-pattern the body's energy field by the use of rhythmic hand motions." C. "The goal is to improve breathing by increasing oxygen to the brain and body tissues." D. "The goal is to decrease blood pressure by body toxin release."

B

A client receiving EMDR therapy says, "After only two sessions of my therapy, I am feeling great. Now I can stop and get on with my life." Which of the following nursing responses is most appropriate? A. "I am thrilled that you have responded so rapidly to EMDR." B. "To achieve lasting results, all eight phases of EMDR must be completed." C. "If I were you, I would complete the EMDR and comply with doctor's orders." D. "How do you feel about continuing the therapy?"

B

A client reports taking St. John's wort for depression. The client states, "I'm taking the recommended dose, but it seems like if two capsules are good, four would be better!" Which is an appropriate nursing reply? A. "Herbal medicines are more likely to cause adverse reactions than prescription medications." B. "Increasing the amount of herbal preparations can lead to overdose and toxicity." C. "The FDA does not regulate herbal remedies, therefore ingredients are often unknown." D. "Certain companies are better than others. Always buy a reputable brand.

B

A client with a history of heavy alcohol use is brought to an emergency department (ED) by family members who state that the client has had nothing to drink in the last 24 hours. Which client symptom should the nurse immediately report to the ED physician? A. Antecubital bruising B. Blood pressure of 180/100 mm Hg C. Mood rating of 2/10 on numeric scale D. Dehydration

B

A despondent client, who has recently lost her husband of 30 years, tearfully states, "I'll feel a lot better if I sell my house and move away." Which nursing reply is most appropriate? A. "I'm confident you know what's best for you." B. "This may not be the best time for you to make such an important decision." C. "Your children will be terribly disappointed." D. "Tell me why you want to make this change."

B

A family member brings the patient to the clinic for a follow-up visit after a stroke. The family member asks the nurse what he can do to decrease his chance of having another stroke. What would be the nurse's best answer? A) "Have your heart checked regularly." B) "Stop smoking as soon as possible." C) "Get medication to bring down your sodium levels." D) "Eat a nutritious diet."

B

A group of military nurses are reviewing the care of victims of biochemical terrorist attacks. The nurses should identify what agents as having the shortest latency? A) Viral agents B) Nerve agents C) Pulmonary agents D) Blood agents

B

A medical nurse educator is reviewing a patients recent episode of metabolic acidosis with members of the nursing staff. What should the educator describe about the role of the kidneys in metabolic acidosis? A) The kidneys retain hydrogen ions and excrete bicarbonate ions to help restore balance. B) The kidneys excrete hydrogen ions and conserve bicarbonate ions to help restore balance. C) The kidneys react rapidly to compensate for imbalances in the body. D) The kidneys regulate the bicarbonate level in the intracellular fluid.

B

A night nurse is reviewing the next days medication administration record (MAR) of a patient who has hemophilia. The nurse notes that the MAR specifies both oral and subcutaneous options for the administration of a PRN antiemetic. What is the nurses best action? A) Ensure that the day nurse knows not to give the antiemetic. B) Contact the prescriber to have the subcutaneous option discontinued. C) Reassess the patients need for antiemetics. D) Remove the subcutaneous route from the patients MAR.

B

A nurse evaluates a clients patient-controlled analgesia (PCA) pump and notices 100 attempts within a 30-minute period. Which is the best rationale for assessing this client for substance use disorder? A. Narcotic pain medication is contraindicated for all clients with active substance-use problems. B. Clients who are regularly using alcohol or benzodiazepines may have developed cross-tolerance to analgesics and require increased doses to achieve effective pain control. C. There is no need to assess the client for substance use disorder. There is an obvious PCA malfunction. D. The client is experiencing symptoms of withdrawal and needs to be accurately assessed for lorazepam (Ativan) dosage.

B

A nurse has taken on the care of a patient who had a coronary artery stent placed yesterday. When reviewing the patients daily medication administration record, the nurse should anticipate administering what drug? A) Ibuprofen B) Clopidogrel C) Dipyridamole D) Acetaminophen

B

A nurse in the ICU receives report from the nurse in the ED about a new patient being admitted with a neck injury he received while diving into a lake. The ED nurse reports that his blood pressure is 85/54, heart rate is 53 beats per minute, and his skin is warm and dry. What does the ICU nurse recognize that that patient is probably experiencing? A) Anaphylactic shock B) Neurogenic shock C) Septic shock D) Hypovolemic shock

B

A nurse is caring for a critically ill patient with autonomic dysreflexia. What clinical manifestations would the nurse expect in this patient? A) Respiratory distress and projectile vomiting B) Bradycardia and hypertension C) Tachycardia and agitation D) Third-spacing and hyperthermia

B

A nurse is caring for a patient diagnosed with a hemorrhagic stroke. When creating this patient's plan of care, what goal should be prioritized? A) Prevent complications of immobility. B) Maintain and improve cerebral tissue perfusion. C) Relieve anxiety and pain. D) Relieve sensory deprivation.

B

A nurse is caring for a patient who experiences debilitating cluster headaches. The patient should be taught to take appropriate medications at what point in the course of the onset of a new headache? A) As soon as the patients pain becomes unbearable B) As soon as the patient senses the onset of symptoms C) Twenty to 30 minutes after the onset of symptoms D) When the patient senses his or her symptoms peaking

B

A nurse is caring for a patient who has been admitted for the treatment of advanced cirrhosis. What assessment should the nurse prioritize in this patient's plan of care? A. Measurement of abdominal girth and body weight B. Assessment for variceal bleeding C. Assessment for signs and symptoms of jaundice D. Monitoring of results of liver function testing

B

A nurse is caring for a patient who has been diagnosed with leukemia. The nurses most recent assessment reveals the presence of ecchymoseson the patients sacral area and petechiae in her forearms. In addition to informing the patients primary care provider, the nurse should perform what action? A) Initiate measures to prevent venous thromboembolism (VTE). B) Check the patients most recent platelet level. C) Place the patient on protective isolation. D) Ambulate the patient to promote circulatory function.

B

A nurse is caring for a patient who is being treated in the hospital for a spontaneous vertebral fracture related to osteoporosis. The nurse should address the nursing diagnosis of Acute Pain Related to Fracture by implementing what intervention? A) Maintenance of high Fowler's positioning whenever possible B) Intermittent application of heat to the patient's back C) Use of a pressure-reducing mattress D) Passive range of motion exercises

B

A nurse is caring for a patient with a blocked bile duct from a tumor. What manifestation of obstructive jaundice should the nurse anticipate? A. Watery, blood-streaked diarrhea B. Orange and foamy urine C. Increased abdominal girth D. Decreased cognition

B

A nurse is caring for a patient with gallstones who has been prescribed ursodeoxycholic acid (UDCA). The patient askshow this medicine is going to help his symptoms. The nurse should be aware of what aspect of this drug's pharmacodynamics? A) It inhibits the synthesis of bile. B) It inhibits the synthesis and secretion of cholesterol. C) It inhibits the secretion of bile. D) It inhibits the synthesis and secretion of amylase.

B

A nurse is caring for patients exposed to a terrorist attack involving chemicals. The nurse has been advised that personal protective equipment must be worn in order to give the highest level of respiratory protection with a lesser level of skin and eye protection. What level protection is this considered? A) Level A B) Level B C) Level C D) Level D

B

A nurse is creating a plan of care for an oncology patient and one of the identified nursing diagnoses is risk for infection related to myelosuppression. What intervention addresses the leading cause of infection-related death in oncology patients? A) Encourage several small meals daily. B) Provide skin care to maintain skin integrity. C) Assist the patient with hygiene, as needed. D) Assess the integrity of the patient's oral mucosa regularly.

B

A nurse is providing discharge education to a patient who has recently been diagnosed with a bleeding disorder. What topic should the nurse prioritize when teaching this patient? A) Avoiding buses, subways, and other crowded, public sites B) Avoiding activities that carry a risk for injury C) Keeping immunizations current D) Avoiding foods high in vitamin K

B

A nurse is reviewing STAT laboratory data of a client presenting in the emergency department. At what minimum blood alcohol level should a nurse expect intoxication to occur? A. 50 mg/dL B. 100 mg/dL C. 250 mg/dL D. 300 mg/dL

B

A nurse is reviewing the trend of a patient's scores on the Glasgow Coma Scale (GCS). This allows the nurse to gauge what aspect of the patient's status? A) Reflex activity B) Level of consciousness C) Cognitive ability D) Sensory involvement

B

A nursing instructor is teaching about complementary therapies. Which student statement indicates that learning has occurred? A. "Complementary therapies view all humans as being biologically similar." B. "Complementary therapies view a person as a combination of multiple, integrated elements." C. "Complementary therapies focus on primarily the structure and functions of the body." D. "Complementary therapies view disease as a deviation from a normal biological state."

B

A participant in a health fair has asked the nurse about the role of drugs in liver disease. What health promotion teaching has the most potential to prevent drug-induced hepatitis? A. Finish all prescribed courses of antibiotics, regardless of symptoms resolution. B. Adhere to dosing recommendations of OTC analgesics. C. Ensure that expired medications are disposed of safely. D. Ensure that pharmacists regularly review drug regimens for potential interactions.

B

A patient converts from normal sinus rhythm at 80 bpm to atrial fibrillation with a ventricular response at 166 bpm. Blood pressure is 162/74 mm Hg. Respiratory rate is 20 breaths per minute with normal chest expansion and clear lungs bilaterally. IV heparin and Cardizem are given. The nurse caring for the patient understands that the main goal of treatment is what? A) Decrease SA node conduction B) Control ventricular heart rate C) Improve oxygenation D) Maintain anticoagulation

B

A patient diagnosed with a pituitary adenoma has arrived on the neurologic unit. When planning the patient's care, the nurse should be aware that the effects of the tumor will primarily depend on what variable? A) Whether the tumor utilizes aerobic or anaerobic respiration B) The specific hormones secreted by the tumor C) The patient's pre-existing health status D) Whether the tumor is primary or the result of metastasi

B

A patient has a diagnosis of multiple myeloma and the nurse is preparing health education in preparation for discharge from the hospital. What action should the nurse promote? A) Daily performance of weight-bearing exercise to prevent muscle atrophy B) Close monitoring of urine output and kidney function C) Daily administration of warfarin (Coumadin) as ordered D) Safe use of supplementary oxygen in t

B

A patient has been admitted to the hospital with a spontaneous vertebral fracture related to osteoporosis. Which of the following nursing diagnoses must be addressed in the plan of care? A) Risk for Aspiration Related to Vertebral Fracture B) Constipation Related to Vertebral Fracture C) Impaired Swallowing Related to Vertebral Fracture D) Decreased Cardiac Output Related to Vertebral Fracture

B

A patient has been admitted to the medical unit for the treatment of Paget's disease. When reviewing the medication administration record, the nurse should anticipate what medications? Select all that apply. A) Calcitonin B) Bisphosphonates C) Alkaline phosphatase D) Calcium gluconate E) Estrogen

B

A patient has experienced an electrical burn and has developed thick eschar over the burn site. Which of the following topical antibacterial agents will the nurse expect the physician to order for the wound? A) Silver sulfadiazine 1% (Silvadene) water-soluble cream B) Mafenide acetate 10% (Sulfamylon) hydrophilic-based cream C) Silver nitrate 0.5% aqueous solution D) Acticoat

B

A patient has just died following urosepsis that progressed to septic shock. The patient's spouse says, I knew this was coming, but I feel so numb and hollow inside. The nurse should know that these statements are characteristic of what? A) Complicated grief and mourning B) Uncomplicated grief and mourning C) Depression stage of dying D) Acceptance stage of dying

B

A patient has recently begun mobilizing during the recovery from an ischemic stroke. To protect the patient's safety during mobilization, the nurse should perform what action? A) Support the patient's full body weight with a waist belt during ambulation. B) Have a colleague follow the patient closely with a wheelchair. C) Avoid mobilizing the patient in the early morning or late evening. D) Ensure that the patient's family members do not participate in mobilization.

B

A patient has returned to the cardiac care unit after having a permanent pacemaker implantation. For which potential complication should the nurse most closely assess this patient? A) Chest pain B) Bleeding at the implantation site C) Malignant hyperthermia D) Bradycardia

B

A patient has returned to the unit after undergoing limb-sparing surgery to remove a metastatic bone tumor. The nurse providing postoperative care in the days following surgery assesses for what complication from surgery? A) Deficient fluid volume B) Delayed wound healing C) Hypocalcemia D) Pathologic fractures

B

A patient has sustained a long bone fracture and the nurse is preparing the patient's care plan. Which of the following should the nurse include in the care plan? A) Administer vitamin D and calcium supplements as ordered. B) Monitor temperature and pulses of the affected extremity. C) Perform passive range of motion exercises as tolerated. D) Administer corticosteroids as ordered.

B

A patient is admitted to the unit with acute cholecystitis. The physician has noted that surgery will be scheduled in 4 days. The patient asks why the surgery is being put off for a week when he has a "sick gallbladder." What rationale would underlie the nurse's response? A) Surgery is delayed until the patient can eat a regular diet without vomiting. B) Surgery is delayed until the acute symptoms subside. C) The patient requires aggressive nutritional support prior to surgery. D) Time is needed to determine whether a laparoscopic procedure can be used.

B

A patient is being treated for bites that she suffered during an assault. After the bites have been examined and documented by a forensic examiner, the nurse should perform what action? A) Apply a dressing saturated with chlorhexidine. B) Wash the bites with soap and water. C) Arrange for the patient to receive a hepatitis B vaccination. D) Assess the patient's immunization history.

B

A patient is being treated in the ICU after a medical error resulted in an acute hemolytic transfusion reaction. What was the etiology of this patient's adverse reaction? A) Antibodies to donor leukocytes remained in the blood. B) The donor blood was incompatible with that of the patient. C) The patient had a sensitivity reaction to a plasma protein in the blood. D) The blood was infused too quickly and overwhelmed the patient's circulatory system

B

A patient is brought to the renal unit from the PACU status post resection of a renal tumor. Which of the following nursing actions should the nurse prioritize in the care of this patient? A) Increasing oral intake B) Managing postoperative pain C) Managing dialysis D) Increasing mobility

B

A patient is receiving the first of two ordered units of PRBCs. Shortly after the initiation of the transfusion, the patient complains of chills and experiences a sharp increase in temperature. What is the nurse's priority action? A) Position the patient in high Fowler's. B) Discontinue the transfusion. C) Auscultate the patient's lungs. D) Obtain a blood specimen from the patient

B

A patient is scheduled for catheter ablation therapy. When describing this procedure to the patients family, the nurse should address what aspect of the treatment? A) Resetting of the hearts contractility B) Destruction of specific cardiac cells C) Correction of structural cardiac abnormalities D) Clearance of partially occluded coronary arteries

B

A patient newly admitted to the telemetry unit is experiencing progressive fatigue, hemoptysis, and dyspnea. Diagnostic testing has revealed that these signs and symptoms are attributable to pulmonary venous hypertension. What valvular disorder should the nurse anticipate being diagnosed in this patient? A) Aortic regurgitation B) Mitral stenosis C) Mitral valve prolapse D) Aortic stenosis

B

A patient newly diagnosed with cancer is scheduled to begin chemotherapy treatment and the nurse is providing anticipatory guidance about potential adverse effects. When addressing the most common adverse effect, what should the nurse describe? A) Pruritis (itching) B) Nausea and vomiting C) Altered glucose metabolism D) Confusio

B

A patient newly diagnosed with thrombocytopenia is admitted to the medical unit. After the admission assessment, the patient asks the nurse to explain the disease. What should the nurse explain to this patient? A) There could be an attack on the platelets by antibodies. B) There could be decreased production of platelets. C) There could be impaired communication between platelets. D) There could be an autoimmune process causing platelet malfunction.

B

A patient on the critical care unit is postoperative day 1 following kidney transplantation from a living donor. The nurse's most recent assessments indicate that the patient is producing copious quantities of dilute urine. What is the nurse's most appropriate response? A) Assess the patient for further signs or symptoms of rejection. B) Recognize this as an expected finding. C) Inform the primary care provider of this finding. D) Administer exogenous antidiuretic hormone as ordered.

B

A patient on the medical unit has a documented history of polycystic kidney disease (PKD). What principle should guide the nurse's care of this patient? A) The disease is self-limiting and cysts usually resolve spontaneously in the fifth or sixth decade of life. B) The patient's disease is incurable and the nurse's interventions will be supportive. C) The patient will eventually require surgical removal of his or her renal cysts. D) The patient is likely to respond favorably to lithotripsy treatment of the cysts.

B

A patient presents at a clinic complaining of pain in his heel so bad that it inhibits his ability to walk. The patient is subsequently diagnosed with plantar fasciitis. This patient's plan of care should include what intervention? A) Wrapping the affected area in lamb's wool or gauze to relieve pressure B) Gently stretching the foot and the Achilles tendon C) Wearing open-toed shoes at all times D) Applying topical analgesic ointment to plantar surface each morning

B

A patient who is being treated in the hospital for a spinal cord injury is advocating for the removal of his urinary catheter, stating that he wants to try to resume normal elimination. What principle should guide the care team's decision regarding this intervention? A) Urinary retention can have serious consequences in patients with SCIs. B) Urinary function is permanently lost following an SCI. C) Urinary catheters should not remain in place for more than 7 days. D) Overuse of urinary catheters can exacerbate nerve damage.

B

A patient who is in shock is receiving dopamine in addition to IV fluids. What principle should inform the nurses care planning during the administration of a vasoactive drug? A) The drug should be discontinued immediately after blood pressure increases. B) The drug dose should be tapered down once vital signs improve. C) The patient should have arterial blood gases drawn every 10 minutes during treatment. D) The infusion rate should be titrated according the patients subjective sensation of adequate perfusion.

B

A patient who was burned in a workplace accident has completed the acute phase of treatment and the plan of care has been altered to prioritize rehabilitation. What nursing action should be prioritized during this phase of treatment? A) Monitoring fluid and electrolyte imbalances B) Providing education to the patient and family C) Treating infection D) Promoting thermoregulation

B

A patient with a history of major depression is brought to the ED by her parents. Which of the following nursing actions is most appropriate? A) Noting that symptoms of physical illness are not relevant to the current diagnosis B) Asking the patient if she has ever thought about taking her own life C) Conducting interviews in a brief and direct manner D) Arranging for the patient to spend time alone to consider her feelings

B

A patient with a longstanding diagnosis of generalized anxiety disorder presents to the emergency room. The triage nurse notes upon assessment that the patient is hyperventilating. The triage nurse is aware that hyperventilation is the most common cause of which acidbase imbalance? A) Respiratory acidosis B) Respiratory alkalosis C) Increased PaCO2 D) CNS disturbances

B

A patient with an occluded coronary artery is admitted and has an emergency percutaneous transluminal coronary angioplasty (PTCA). The patient is admitted to the cardiac critical care unit after the PTCA. For what complication should the nurse most closely monitor the patient? A) Hyperlipidemia B) Bleeding at insertion site C) Left ventricular hypertrophy D) Congestive heart failure

B

A patient with cirrhosis has experienced a progressive decline in his health; and liver transplantation is being considered by the interdisciplinary team. How will the patient's prioritization for receiving a donor liver be determined? A. By considering that patient's age and prognosis B. By objectively determining the patient's medical need C. By objectively assessing the patient's willingness to adhere to post-transplantation care D. By systematically ruling out alternative treatment options

B

A patient with diabetes has been diagnosed with osteomyelitis. The nurse notes that the patient's right foot is pale and mottled, cool to touch, with a capillary refill of greater than 3 seconds. The nurse should suspect what type of osteomyelitis? A) Hematogenous osteomyelitis B) Osteomyelitis with vascular insufficiency C) Contiguous-focus osteomyelitis D) Osteomyelitis with muscular deterioration

B

A patient with diabetes is attending a class on the prevention of associated diseases. What action should the patient perform to reduce the risk of osteomyelitis? A) Increase calcium and vitamin intake. B) Perform meticulous foot care. C) Exercise 3 to 4 times weekly for at least 30 minutes. D) Take corticosteroids as ordered.

B

A patient with pericarditis has just been admitted to the CCU. The nurse planning the patients care should prioritize what nursing diagnosis? A) Anxiety related to pericarditis B) Acute pain related to pericarditis C) Ineffective tissue perfusion related to pericarditis D) Ineffective breathing pattern related to pericarditis

B

A patient's most recent diagnostic imaging has revealed that his lung cancer has metastasized to his bones and liver. What is the most likely mechanism by which the patient's cancer cells spread? A) Hematologic spread B) Lymphatic circulation C) Invasion D) Angiogenesis

B

A patients blood work reveals a platelet level of 17,000/mm3. When inspecting the patients integumentary system, what finding would be most consistent with this platelet level? A) Dermatitis B) Petechiae C) Urticaria D) Alopecia

B

A patients burns have required a homograft. During the nurses most recent assessment, the nurse observes that the graft is newly covered with purulent exudate. What is the nurses most appropriate response? A) Perform mechanical dbridement to remove the exudate and prevent further infection. B) Inform the primary care provider promptly because the graft may need to be removed. C) Perform range of motion exercises to increase perfusion to the graft site and facilitate healing. D) Document this finding as an expected phase of graft healing.

B

A preceptor is discussing stroke with a new nurse on the unit. The preceptor would tell the new nurse which cardiac dysrhythmia is associated with cardiogenic embolic strokes? A) Ventricular tachycardia B) Atrial fibrillation C) Supraventricular tachycardia D) Bundle branch block

B

A rehabilitation nurse caring for a patient who has had a stroke is approached by the patient's family and asked why the patient has to do so much for herself when she is obviously struggling. What would be the nurse's best answer? A) "We are trying to help her be as useful as she possibly can." B) "The focus on care in a rehabilitation facility is to help the patient to resume as much self-care as possible." C) "We aren't here to care for her the way the hospital staff did; we are here to help her get better so she can go home." D) "Rehabilitation means helping patients do exactly what they did before their stroke."

B

An adult patient is admitted to the ED with chest pain. The patient states that he had developed unrelieved chest pain that was present for approximately 20 minutes before coming to the hospital. To minimize cardiac damage, the nurse should expect to administer which of the following interventions? A) Thrombolytics, oxygen administration, and nonsteroidal anti-inflammatories B) Morphine sulphate, oxygen, and bed rest C) Oxygen and beta-adrenergic blockers D) Bed rest, albuterol nebulizer treatments, and oxygen

B

An industrial site has experienced a radiation leak and workers who have been potentially affected are en route to the hospital. To minimize the risks of contaminating the hospital, managers should perform what action? A) Place all potential victims on reverse isolation. B) Establish a triage outside the hospital. C) Have hospital staff put on personal protective equipment. D) Place hospital staff on abbreviated shifts of no more than 4 hours.

B

An older adult patient has fallen in her home and is brought to the emergency department by ambulance with a suspected fractured hip. X-rays confirm a fracture of the left femoral neck. When planning assessments during the patient's presurgical care, the nurse should be aware of the patient's heightened risk of what complication? A) Osteomyelitis B) Avascular necrosis C) Phantom pain D) Septicemia

B

An oncology nurse educator is providing health education to a patient who has been diagnosed with skin cancer. The patient's wife has asked about the differences between normal cells and cancer cells. What characteristic of a cancer cell should the educator cite? A) Malignant cells contain more fibronectin than normal body cells. B) Malignant cells contain proteins called tumor-specific antigens. C) Chromosomes contained in cancer cells are more durable and stable than those of normal cells. D) The nuclei of cancer cells are unusually large, but regularly shaped.

B

As the American population ages, nurses expect see more patients admitted to long- term care facilities in need of palliative care. Regulations now in place that govern how the care in these facilities is both organized and reimbursed emphasize what aspect of care? A) Ongoing acute care B) Restorative measures C) Mobility and socialization D) Incentives to palliative care

B

New nurses on the telemetry unit have been paired with preceptors. One new nurse asks her preceptor to explain depolarization. What would be the best answer by the preceptor? A) Depolarization is the mechanical contraction of the heart muscles. B) Depolarization is the electrical stimulation of the heart muscles. C) Depolarization is the electrical relaxation of the heart muscles. D) Depolarization is the mechanical relaxation of the heart muscle

B

Patients who are enrolled in hospice care through Medicare are often felt to suffer unnecessarily because they do not receive adequate attention for their symptoms of the underlying illness. What factor most contributes to this phenomenon? A) Unwillingness to overmedicate the dying patient B) Rules concerning completion of all cure-focused medical treatment C) Unwillingness of patients and families to acknowledge the patient is terminal D) Lack of knowledge of patients and families regarding availability of care

B

The acute medical nurse is preparing to wean a patient from the ventilator. Which assessment parameter is most important for the nurse to assess? A) Fluid intake for the last 24 hours B) Baseline arterial blood gas (ABG) levels C) Prior outcomes of weaning D) Electrocardiogram (ECG) results

B

The critical care nurse is caring for a patient who has had an MI. The nurse should expect to assist with establishing what hemodynamic monitoring procedure to assess the patients left ventricular function? A) Central venous pressure (CVP) monitoring B) Pulmonary artery pressure monitoring (PAPM) C) Systemic arterial pressure monitoring (SAPM) D) Arterial blood gases (ABG)

B

The critical care nurse is monitoring the patient's urine output and drains following renal surgery. What should the nurse promptly report to the physician? A) Increased pain on movement B) Absence of drain output C) Increased urine output D) Blood-tinged serosanguineous drain outpu

B

The nurse caring for a patient post colon resection is assessing the patient on the second postoperative day. The nasogastric tube (NG) remains patent and continues at low intermittent wall suction. The IV is patent and infusing at 125 mL/hr. The patient reports pain at the incision site rated at a 3 on a 0-to-10 rating scale. During your initial shift assessment, the patient complains of cramps in her legs and a tingling sensation in her feet. Your assessment indicates decreased deep tendon reflexes (DTRs) and you suspect the patient has hypokalemia. What other sign or symptom would you expect this patient to exhibit? A) Diarrhea B) Dilute urine C) Increased muscle tone D) Joint pain

B

The nurse caring for a patient receiving a transfusion notes that 15 minutes after the infusion of packed red blood cells (PRBCs) has begun, the patient is having difficulty breathing and complains of severe chest tightness. What is the most appropriate initial action for the nurse to take? A) Notify the patient's physician. B) Stop the transfusion immediately. C) Remove the patient's IV access. D) Assess the patient's chest sounds and vital signs.

B

The nurse coming on shift on the medical unit is taking a report on four patients. What patient does the nurse know is at the greatest risk of developing ESKD? A) A patient with a history of polycystic kidney disease B) A patient with diabetes mellitus and poorly controlled hypertension C) A patient who is morbidly obese with a history of vascular disorders D) A patient with severe chronic obstructive pulmonary disease

B

The nurse is analyzing a rhythm strip. What component of the ECG corresponds to the resting state of the patients heart? A) P wave B) T wave C) U wave D) QRS complex

B

The nurse is assessing a patient who is known to have right-sided HF. What assessment finding is most consistent with this patients diagnosis? A) Pulmonary edema B) Distended neck veins C) Dry cough D) Orthopnea

B

The nurse is caring for a patient diagnosed with an ischemic stroke and knows that effective positioning of the patient is important. Which of the following should be integrated into the patient's plan of care? A) The patient's hip joint should be maintained in a flexed position. B) The patient should be in a supine position unless ambulating. C) The patient should be placed in a prone position for 15 to 30 minutes several times a day. D) The patient should be placed in a Trendelenberg position two to three times daily to promote cerebral perfusion.

B

The nurse is caring for a patient recovering from an ischemic stroke. What intervention best addresses a potential complication after an ischemic stroke? A) Providing frequent small meals rather than three larger meals B) Teaching the patient to perform deep breathing and coughing exercises C) Keeping a urinary catheter in situ for the full duration of recovery D) Limiting intake of insoluble fiber

B

The nurse is caring for a patient suspected of having ARDS. What is the most likely diagnostic test ordered in the early stages of this disease to differentiate the patients symptoms from those of a cardiac etiology? A) Carboxyhemoglobin level B) Brain natriuretic peptide (BNP) level C) C-reactive protein (CRP) level D) Complete blood coun

B

The nurse is caring for a patient who is believed to have just experienced an MI. The nurse notes changes in the ECG of the patient. What change on an ECG most strongly suggests to the nurse that ischemia is occurring? A) P wave inversion B) T wave inversion C) Q wave changes with no change in ST or T wave D) P wave enlargement

B

The nurse is caring for a patient who is scheduled to undergo mechanical valve replacement. Patient education should include which of the following? A) Use of patient-controlled analgesia B) Long-term anticoagulant therapy C) Steroid therapy D) Use of IV diuretics

B

The nurse is caring for a patient with an advanced stage of breast cancer and the patient has recently learned that her cancer has metastasized. The nurse enters the room and finds the patient struggling to breath and the nurse's rapid assessment reveals that the patient's jugular veins are distended. The nurse should suspect the development of what oncologic emergency? A) Increased intracranial pressure B) Superior vena cava syndrome (SVCS) C) Spinal cord compression D) Metastatic tumor of the neck

B

The nurse is participating in the care of a patient with increased ICP. What diagnostic test is contraindicated in this patients treatment? A) Computed tomography (CT) scan B) Lumbar puncture C) Magnetic resonance imaging (MRI) D) Venous Doppler studies

B

The nurse is reviewing the echocardiography results of a patient who has just been diagnosed with dilated cardiomyopathy (DCM). What changes in heart structure characterize DCM? A) Dilated ventricles with atrophy of the ventricles B) Dilated ventricles without hypertrophy of the ventricles C) Dilation and hypertrophy of all four heart chambers D) Dilation of the atria and hypertrophy of the ventricles

B

The nurse manager in the ED receives information that a local chemical plant has had a chemical leak. This disaster is assigned a status of level II. What does this classification indicate? A) First responders can manage the situation. B) Regional efforts and aid from surrounding communities can manage the situation. C) Statewide or federal assistance is required. D) The area must be evacuated immediately.

B

The nurse observes that the family members of a patient who was injured in an accident are blaming each other for the circumstances leading up to the accident. The nurse appropriately lets the family members express their feelings of responsibility, while explaining that there was probably little they could do to prevent the injury. In what stage of crisis is this family? A) Anxiety and denial B) Remorse and guilt C) Anger D) Grief

B

The nurse overseeing care in the ICU reviews the shift report on four patients. The nurse recognizes which patient to be at greatest risk for the development of cardiogenic shock? A) The patient admitted with acute renal failure B) The patient admitted following an MI C) The patient admitted with malignant hypertension D) The patient admitted following a stroke

B

The nursing educator is reviewing the signs and symptoms of heat stroke with a group of nurses who provide care in a desert region. The educator should describe what sign or symptom? A) Hypertension with a wide pulse pressure B) Anhidrosis C) Copious diuresis D) Cheyne-Stokes respirations

B

The orthopedic nurse should assess for signs and symptoms of Volkmann's contracture if a patient has fractured which of the following bones? A) Femur B) Humerus C) Radial head D) Clavicle

B

The physician has ordered continuous positive airway pressure (CPAP) with the delivery of a patients high-flow oxygen therapy. The patient asks the nurse what the benefit of CPAP is. What would be the nurses best response? A) CPAP allows a higher percentage of oxygen to be safely used. B) CPAP allows a lower percentage of oxygen to be used with a similar effect. C) CPAP allows for greater humidification of the oxygen that is administered. D) CPAP allows for the elimination of bacterial growth in oxygen delivery systems.

B

The staff educator is presenting a workshop on valvular disorders. When discussing the pathophysiology of aortic regurgitation the educator points out the need to emphasize that aortic regurgitation causes what? A) Cardiac tamponade B) Left ventricular hypertrophy C) Right-sided heart failure D) Ventricular insufficiency

B

What would the critical care nurse recognize as a condition that may indicate a patient's need to have a tracheostomy? A) A patient has a respiratory rate of 10 breaths per minute. B) A patient requires permanent ventilation. C) A patient exhibits symptoms of dyspnea. D) A patient has respiratory acidosis.

B

Which of the following patients should the nurse recognize as being at the highest risk for the development of osteomyelitis? A) A middle-age adult who takes ibuprofen daily for rheumatoid arthritis B) An elderly patient with an infected pressure ulcer in the sacral area C) A 17-year-old football player who had orthopedic surgery 6 weeks prior D) An infant diagnosed with jaundice

B

While completing a health history on a patient who has recently experienced a seizure, the nurse would assess for what characteristic associated with the postictal state? A) Epileptic cry B) Confusion C) Urinary incontinence D) Body rigidity

B

You are caring for a patient with a diagnosis of pancreatitis. The patient was admitted from a homeless shelter and is a vague historian. The patient appears malnourished and on day 3 of the patients admission total parenteral nutrition (TPN) has been started. Why would you know to start the infusion of TPN slowly? A) Patients receiving TPN are at risk for hypercalcemia if calories are started too rapidly. B) Malnourished patients receiving parenteral nutrition are at risk for hypophosphatemia if calories are started too aggressively. C) Malnourished patients who receive fluids too rapidly are at risk for hypernatremia. D) Patients receiving TPN need a slow initiation of treatment in order to allow digestive enzymes to accumulate

B

You are the surgical nurse caring for a 65-year-old female patient who is postoperative day 1 following a thyroidectomy. During your shift assessment, the patient complains of tingling in her lips and fingers. She tells you that she has an intermittent spasm in her wrist and hand and she exhibits increased muscle tone. What electrolyte imbalance should you first suspect? A) Hypophosphatemia B) Hypocalcemia C) Hypermagnesemia D) Hyperkalemia

B

A 67-year-old woman experienced the death of her husband from a sudden myocardial infarction 5 weeks ago. The nurse recognizes that the woman will be going through the process of mourning for an extended period of time. What processes of mourning will allow the woman to accommodate the loss in a healthy way? Select all that apply. A) Reiterating her anger at her husband's care team B) Reinvesting in new relationships at the appropriate time C) Reminiscing about the relationship she had with her husband D) Relinquishing old attachments to her husband at the appropriate time E) Renewing her lifelong commitment to her husband

B,C,D

A critical care nurse is planning assessments in the knowledge that patients in shock are vulnerable to developing fluid replacement complications. For what signs and symptoms should the nurse monitor the patient? Select all that apply. A) Hypovolemia B) Difficulty breathing C) Cardiovascular overload D) Pulmonary edema E) Hypoglycemia

B,C,D

A patient is admitted to the ED complaining of abdominal pain. Further assessment of the abdomen reveals signs of peritoneal irritation. What assessment findings would corroborate this diagnosis? Select all that apply. A) Ascites B) Rebound tenderness C) Changes in bowel sounds D) Muscular rigidity E) Copious diarrhea

B,C,D

The critical care nurse is caring for a patient who is in cardiogenic shock. What assessments must the nurse perform on this patient? Select all that apply. A) Platelet level B) Fluid status C) Cardiac rhythm D) Action of medications E) Sputum volume

B,C,D

The nurse is caring for a patient who is rapidly progressing toward brain death. The nurse should be aware of what cardinal signs of brain death? Select all that apply. A) Absence of pain response B) Apnea C) Coma D) Absence of brain stem reflexes E) Absence of deep tendon reflexes

B,C,D

A family asks the nurse why their son was diagnosed with PTSD and others in the accident were not. Which of the following information should the nurse offer? (Select all that apply.) A. An individual's religious affiliation can affect response to trauma. B. Responses are affected by how an individual handled previous trauma. C. Protectiveness of family and friends can help an individual deal with trauma. D. Control over the possibility of recurrence can affect the response to trauma. E. The time in which the trauma occurred can affect the individual's response.

B,C,D,E

The nurse is caring for a patient with acute glomerular inflammation. When assessing for the characteristic signs and symptoms of this health problem, the nurse should include which assessments? Select all that apply. A) Percuss for pain in the right lower abdominal quadrant. B) Assess for the presence of peripheral edema. C) Auscultate the patient's apical heart rate for dysrhythmias. D) Assess the patient's BP. E) Assess the patient's orientation and judgment.

B,D

A client diagnosed with PTSD states, "Why did my doctor prescribe an antidepressant rather than an antianxiety drug for me?" Which of the following are the most appropriate nursing responses? (Select all that apply.) A. "I'm not sure, because antianxiety drugs have been approved by the FDA for PTSD." B. "Antidepressants are now considered first-line treatment choice for PTSD." C. "Many people have adverse reactions to antianxiety drugs." D. "Because of their addictive properties, antianxiety drugs are less desirable." E. "There have been no controlled studies on the effect of antianxiety drugs on PTSD."

B,D,E

A patient is brought to the ED by ambulance after a motor vehicle accident in which the patient received blunt trauma to the chest. The patient is in acute respiratory failure, is intubated, and is transferred to the ICU. What parameters of care should the nurse monitor most closely? Select all that apply. A) Coping B) Level of consciousness C) Oral intake D) Arterial blood gases E) Vital signs

B,D,E

A patient has been admitted to the critical care unit with a diagnosis of toxic hepatitis. When planning the patient's care, the nurse should be aware of what potential clinical course of this health problem? Place the following events in the correct sequence. 1. Fever rises 2. Hematemesis 3. Clotting abnormalities 4. Vascular collapse 5. Coma A. 1,2,5,4,3 B. 1,2,3,4,5 C. 2,3,1,4,5 D. 3,1,2,5,4

B. 1,2,3,4,5

A nurse in a busy emergency department provides care for many patients who present with contusions, strains, or sprains. Treatment modalities that are common to all of these musculoskeletal injuries include which of the following? Select all that apply. A) Massage B) Applying ice C) Compression dressings D) Resting the affected extremity E) Corticosteroids F) Elevating the injured limb

B. C,D,F

. A patient is receiving care in the intensive care unit for acute pancreatitis. The nurse is aware that pancreatic necrosis is a major cause of morbidity and mortality in patients with acute pancreatitis. Consequently, the nurse should assess for what signs or symptoms of this complication? A) Sudden increase in random blood glucose readings B) Increased abdominal girth accompanied by decreased level of consciousness C) Fever, increased heart rate and decreased blood pressure D) Abdominal pain unresponsive to analgesics

C

A 17-year-old boy is being treated in the ICU after going into cardiac arrest during a football practice. Diagnostic testing reveals cardiomyopathy as the cause of the arrest. What type of cardiomyopathy is particularly common among young people who appear otherwise healthy? A) Dilated cardiomyopathy (DCM). B) Arrhythmogenic right ventricular cardiomyopathy (ARVC) C) Hypertrophic cardiomyopathy (HCM) D) Restrictive or constrictive cardiomyopathy (RCM)

C

A 20 year-old is brought in by ambulance to the emergency department after being involved in a motorcycle accident. The patient has an open fracture of his tibia. The wound is highly contaminated and there is extensive soft-tissue damage. How would this patient's fracture likely be graded? A) Grade I B) Grade II C) Grade III D) Grade IV

C

A client with chronic lower back pain states, "My nurse practitioner told me that acupuncture may enhance the effect of the medications and physical therapy prescribed." What type of therapy is the nurse practitioner recommending? A. Alternative therapy B. Physiotherapy C. Complementary therapy D. Biopsychosocial therapy

C

A client with hypertension has been prescribed hydrochlorothiazide. What nursing action will best reduce the client's risk for electroyte disturbances? A. Maintain a low sodium diet B. Encourage the use of over the counter calcium supplements C. Ensure the client has sufficient potassium intake D. Encourage fluid intake

C

A client with several chronic health problems has been newly diagnosed with a qualitative platelet defect. What component of the patients previous medication regimen may have contributed to the development of this disorder? A) Calcium carbonate B) Vitamin B12 C) Aspirin D) Vitamin D

C

A group of disaster survivors is working with the critical incident stress management (CISM) team. Members of this team should be guided by what goal? A) Determining whether the incident was managed effectively B) Educating survivors on potential coping strategies for future disasters C) Providing individuals with education about recognizing stress reactions D) Determining if individuals responded appropriately during the incident

C

A nurse at a long-term care facility is amending the care plan of a resident who has just been diagnosed with essential thrombocythemia (ET). The nurse should anticipate the administration of what medication? A) Dalteparin B) Allopurinol C) Hydroxyurea D) HydrochlorothiazideV

C

A nurse is caring for a patient who has suffered a hip fracture and who will require an extended hospital stay. The nurse should ensure that the patient does which of the following in order to prevent common complications associated with a hip fracture? A) Avoid requesting analgesia unless pain becomes unbearable. B) Use supplementary oxygen when transferring or mobilizing. C) Increase fluid intake and perform prescribed foot exercises. D) Remain on bed rest for 14 days or until instructed by the orthopedic surgeon.

C

A nurse is caring for a patient with Paget's disease and is reviewing the patient's most recent laboratory values. Which of the following values is most characteristic of Paget's disease? A) An elevated level of parathyroid hormone and low calcitonin levels B) A low serum alkaline phosphatase level and a low serum calcium level C) An elevated serum alkaline phosphatase level and a normal serum calcium level D) An elevated calcitonin level and low levels of parathyroid hormone

C

A nurse is creating a care plan for a patient with acute pancreatitis. The care plan includes reduced activity. What rationale for this intervention should be cited in the care plan? A) Bed rest reduces the patient's metabolism and reduces the risk of metabolic acidosis. B) Reduced activity protects the physical integrity of pancreatic cells. C) Bed rest lowers the metabolic rate and reduces enzyme production. D) Inactivity reduces caloric need and gastrointestinal motility.

C

A nurse is discussing conservative management of tendonitis with a patient. Which of the following may be an effective approach to managing tendonitis? A) Weight reduction B) Use of oral opioid analgesics C) Intermittent application of ice and heat D) Passive range of motion exercises

C

A nurse is giving an educational class to members of the local disaster team. What should the nurse instruct members of the disaster team to do in a chemical bioterrorist attack? A) Cover their eyes. B) Put on a personal protective equipment mask. C) Stand up. D) Crawl to an exit.

C

A nurse is performing a shift assessment on an elderly patient who is recovering after surgery for a hip fracture. The nurse notes that the patient is complaining of chest pain, has an increased heart rate, and increased respiratory rate. The nurse further notes that the patient is febrile and hypoxic, coughing, and producing large amounts of thick, white sputum. The nurse recognizes that this is a medical emergency and calls for assistance, recognizing that this patient is likely demonstrating symptoms of what complication? A) Avascular necrosis of bone B) Compartment syndrome C) Fat embolism syndrome D) Complex regional pain syndrome

C

A nurse is performing an admission assessment of a patient with a diagnosis of cirrhosis. What technique should the nurse use to palpate the patient's liver? A. Place hand under the right lower abdominal quadrant and press down lightly with the other hand. B. Place the left hand over the abdomen and behind the left side at the 11th rib. C. Place hand under right lower rib cage and press down lightly with the other hand. D. Hold hand 90 degree to right side of the abdomen and push down firmly.

C

A nurse is reviewing the pathophysiology that may underlie a patient's decreased bone density. What hormone should the nurse identify as inhibiting bone resorption and promoting bone formation? A) Estrogen B) Parathyroid hormone (PTH) C) Calcitonin D) Progesterone

C

A nurse who provides care on a burn unit is preparing to apply a patients ordered topical antibiotic ointment. What action should the nurse perform when administering this medication? A) Apply the new ointment without disturbing the existing layer of ointment. B) Apply the ointment using a sterile tongue depressor. C) Apply a layer of ointment approximately 1/16 inch thick. D) Gently irrigate the wound bed after applying the antibiotic ointment.

C

A nursing educator is reviewing the risk factors for osteoporosis with a group of recent graduates. What risk factor of the following should the educator describe? A) Recurrent infections and prolonged use of NSAIDs B) High alcohol intake and low body mass index C) Small frame, female gender, and Caucasian ethnicity D) Male gender, diabetes, and high protein intake

C

A patient diagnosed with a hemorrhagic stroke has been admitted to the neurologic ICU. The nurse knows that teaching for the patient and family needs to begin as soon as the patient is settled on the unit and will continue until the patient is discharged. What will family education need to include? A) How to differentiate between hemorrhagic and ischemic stroke B) Risk factors for ischemic stroke C) How to correctly modify the home environment D) Techniques for adjusting the patient's medication dosages at home

C

A patient diagnosed with transient ischemic attacks (TIAs) is scheduled for a carotid endarterectomy. The nurse explains that this procedure will be done for what purpose? A) To decrease cerebral edema B) To prevent seizure activity that is common following a TIA C) To remove atherosclerotic plaques blocking cerebral flow D) To determine the cause of the TIA

C

A patient experienced a 33% TBSA burn 72 hours ago. The nurse observes that the patients hourly urine output has been steadily increasing over the past 24 hours. How should the nurse best respond to this finding? A) Obtain an order to reduce the rate of the patients IV fluid infusion. B) Report the patients early signs of acute kidney injury (AKI). C) Recognize that the patient is experiencing an expected onset of diuresis. D) Administer sodium chloride as ordered to compensate for this fluid loss.

C

A patient has a glomerular filtration rate (GFR) of 43 mL/min/1.73 m2. Based on this GFR, the nurse interprets that the patient's chronic kidney disease is at what stage? A) Stage 1 B) Stage 2 C) Stage 3 D) Stage 4

C

A patient has come to the orthopedic clinic for a follow-up appointment 6 weeks after fracturing his ankle. Diagnostic imaging reveals that bone union is not taking place. What factor may have contributed to this complication? A) Inadequate vitamin D intake B) Bleeding at the injury site C) Inadequate immobilization D) Venous thromboembolism (VTE)

C

A patient has presented to the emergency department with an injury to the wrist. The patient is diagnosed with a third-degree strain. Why would the physician order an x-ray of the wrist? A) Nerve damage is associated with third-degree strains. B) Compartment syndrome is associated with third-degree strains. C) Avulsion fractures are associated with third-degree strains. D) Greenstick fractures are associated with third-degree strains.

C

A patient has undergone a laparoscopic cholecystectomy and is being prepared for discharge home. When providing health education, the nurse should prioritize which of the following topics? A) Management of fluid balance in the home setting B) The need for blood glucose monitoring for the next week C) Signs and symptoms of intra-abdominal complications D) Appropriate use of prescribed pancreatic enzymes

C

A patient is admitted to the ICU with acute pancreatitis. The patient's family asks what causes acute pancreatitis. The critical care nurse knows that a majority of patients with acute pancreatitis have what? A) Type 1 diabetes B) An impaired immune system C) Undiagnosed chronic pancreatitis D) An amylase deficiency

C

A patient is brought to the ED by ambulance after swallowing highly acidic toilet bowl cleaner 2 hours earlier. The patient is alert and oriented. What is the care team's most appropriate treatment? A) Administering syrup of ipecac B) Performing a gastric lavage C) Giving milk to drink D) Referring to psychiatry

C

A patient is undergoing diagnostic testing for mitral stenosis. What statement by the patient during the nurses interview is most suggestive of this valvular disorder? A) I get chest pain from time to time, but it usually resolves when I rest. B) Sometimes when Im resting, I can feel my heart skip a beat. C) Whenever I do any form of exercise I get terribly short of breath. D) My feet and ankles have gotten terribly puffy the last few weeks

C

A patient on the oncology unit is receiving carmustine, a chemotherapy agent, and the nurse is aware that a significant side effect of this medication is thrombocytopenia. Which symptom should the nurse assess for in patients at risk for thrombocytopenia? A) Interrupted sleep pattern B) Hot flashes C) Epistaxis (nose bleed) D) Increased weight

C

A patient with a diagnosis of cirrhosis has developed variceal bleeding and will imminently undergo variceal banding. What psychosocial nursing diagnosis should the nurse most likely prioritize during this phase of the patient's treatment? A. Decisional Conflict B. Deficient Knowledge C. Death Anxiety D. Disturbed Thought Processes

C

A patient with a history of atrial fibrillation has contacted the clinic saying that she has accidentally overdosed on her prescribed warfarin (Coumadin). The nurse should recognize the possible need for what antidote? A) IVIG B) Factor X C) Vitamin K D) Factor VIII

C

A patient with a myelodysplastic syndrome is being treated on the medical unit. What assessment finding should prompt the nurse to contact the patients primary care provider? A) The patient is experiencing a frontal lobe headache. B) The patient has an episode of urinary incontinence. C) The patient has an oral temperature of 37.5C (99.5F). D) The patients SpO2 is 91% on room air.

C

A patient with a new diagnosis of ischemic stroke is deemed to be a candidate for treatment with tissue plasminogen activator (t-PA) and has been admitted to the ICU. In addition to closely monitoring the patient's cardiac and neurologic status, the nurse monitors the patient for signs of what complication? A) Acute pain B) Septicemia C) Bleeding D) Seizures

C

A patient with a partial-thickness burn injury had Biobrane applied 2 weeks ago. The nurse notices that the Biobrane is separating from the burn wound. What is the nurses most appropriate intervention? A) Reinforce the Biobrane dressing with another piece of Biobrane. B) Remove the Biobrane dressing and apply a new dressing. C) Trim away the separated Biobrane. D) Notify the physician for further emergency-related orders.

C

A patient with end-stage heart failure has participated in a family meeting with the interdisciplinary team and opted for hospice care. On what belief should the patient's care in this setting be based? A) Meaningful living during terminal illness requires technologic interventions. B) Meaningful living during terminal illness is best supported in designated facilities. C) Meaningful living during terminal illness is best supported in the home. D) Meaningful living during terminal illness is best achieved by prolonging physiologic dying

C

A patient with hypertrophic cardiomyopathy (HCM) has been admitted to the medical unit. During the nurses admission interview, the patient states that she takes over-the-counter water pills on a regular basis. How should the nurse best respond to the fact that the patient has been taking diuretics? A) Encourage the patient to drink at least 2 liters of fluid daily. B) Increase the patients oral sodium intake. C) Inform the care provider because diuretics are contraindicated. D) Ensure that the patients fluid balance is monitored vigilantly.

C

A patient with mitral stenosis exhibits new symptoms of a dysrhythmia. Based on the pathophysiology of this disease process, the nurse would expect the patient to exhibit what heart rhythm? A) Ventricular fibrillation (VF) B) Ventricular tachycardia (VT) C) Atrial fibrillation D) Sinus bradycardia

C

A patient with multiple trauma is brought to the ED by ambulance after a fall while rock climbing. What is a responsibility of the ED nurse in this patient's care? A) Intubating the patient B) Notifying family members C) Ensuring IV access D) Delivering specimens to the laboratory

C

A public health nurse has formed an interdisciplinary team that is developing an educational program entitled Cancer: The Risks and What You Can Do About Them. Participants will receive information, but the major focus will be screening for relevant cancers. This program is an example of what type of health promotion activity? A) Disease prophylaxis B) Risk reduction C) Secondary prevention D) Tertiary prevention

C

A school nurse is assessing a student who was kicked in the shin during a soccer game. The area of the injury has become swollen and discolored. The triage nurse recognizes that the patient has likely sustained what? A) Sprain B) Strain C) Contusion D) Dislocation

C

A young man with a diagnosis of hemophilia A has been brought to emergency department after suffering a workplace accident resulting in bleeding. Rapid assessment has revealed the source of the patients bleeding and established that his vital signs are stable. What should be the nurses next action? A) Position the patient in a prone position to minimize bleeding. B) Establish IV access for the administration of vitamin K. C) Prepare for the administration of factor VIII. D) Administer a normal saline bolus to increase circulatory volume.

C

A young patient is being treated for a femoral fracture suffered in a snowboarding accident. The nurse's most recent assessment reveals that the patient is uncharacteristically confused. What diagnostic test should be performed on this patient? A) Electrolyte assessment B) Electrocardiogram C) Arterial blood gases D) Abdominal ultrasound

C

An emergency department nurse is triaging a 77-year-old man who presents with uncharacteristic fatigue as well as back and rib pain. The patient denies any recent injuries. The nurse should recognize the need for this patient to be assessed for what health problem? A) Hodgkin disease B) Non-Hodgkin lymphoma C) Multiple myeloma D) Acute thrombocythemia

C

Both situational and intrapersonal factors most likely contribute to an individual's stress response. Which factor would a nurse categorize as intrapersonal? 1. Occupational opportunities 2. Economic conditions 3. Degree of flexibility 4. Availability of social supports

C

Diagnostic imaging reveals that the quantity of fluid in a clients pericardial sac is dangerously increased. The nurse should collaborate with the other members of the care team to prevent the development of what complication? A) Pulmonary edema B) Pericardiocentesis C) Cardiac tamponade D) Pericarditis

C

During a CPR class, a participant asks about the difference between cardioversion and defibrillation. What would be the instructors best response? A) Cardioversion is done on a beating heart; defibrillation is not. B) The difference is the timing of the delivery of the electric current. C) Defibrillation is synchronized with the electrical activity of the heart, but cardioversion is not. D) Cardioversion is always attempted before defibrillation because it has fewer risks.

C

During a client's care conference, the team is discussing whether the client is a candidate for surgery maze procedure. What would be the most important criterion for a client to have this surgery? A. Angina pectoris not responsive to other treatments. B. Decreased activity tolerance related to decreased cardiac output C. Refractory atrial fibrillation D. Ventricular fibrillation not responsive to other treatment

C

Herbs and plants can be useful in treating a variety of conditions. Which herbal treatment should a nurse determine as appropriate for a client experiencing frequent migraine headaches? A. Saint John's wort combined with an antidepressant B. Ginger root combined with a beta-blocker C. Feverfew, used according to directions D. Kava-kava added to a regular diet

C

On the first day of a clients alcohol detoxification, which nursing intervention should take priority? A. Strongly encourage the client to attend 90 Alcoholics Anonymous meetings in 90 days. B. Educate the client about the biopsychosocial consequences of alcohol abuse. C. Administer ordered chlordiazepoxide (Librium) in a dosage according to protocol. D. Administer vitamin B1 to prevent Wernicke-Korsakoff syndrome.

C

Six weeks after an above-the-knee amputation (AKA), a patient returns to the outpatient office for a routine postoperative checkup. During the nurse's assessment, the patient reports symptoms of phantom pain. What should the nurse tell the patient to do to reduce the discomfort of the phantom pain? A) Apply intermittent hot compresses to the area of the amputation. B) Avoid activity until the pain subsides. C) Take opioid analgesics as ordered. D) Elevate the level of the amputation site.

C

The ICU nurse is caring for a patient in neurogenic shock following an overdose of antianxiety medication. When assessing this patient, the nurse should recognize what characteristic of neurogenic shock? A) Hypertension B) Cool, moist skin C) Bradycardia D) Signs of sympathetic stimulation

C

The acute care nurse is providing care for an adult patient who is in hypovolemic shock. The nurse recognizes that antidiuretic hormone (ADH) plays a significant role in this health problem. What assessment finding will the nurse likely observe related to the role of the ADH during hypovolemic shock? A) Increased hunger B) Decreased thirst C) Decreased urinary output D) Increased capillary perfusion

C

The current phase of a patients treatment for a burn injury prioritizes wound care, nutritional support, and prevention of complications such as infection. Based on these care priorities, the patient is in what phase of burn care? A) Emergent B) Immediate resuscitative C) Acute D) Rehabilitation

C

The intensive care nurse caring for a patient in shock is planning assessments and interventions related to the patients nutritional needs. What physiologic process contributes to these increased nutritional needs? A) The use of albumin as an energy source by the body because of the need for increased adenosine triphosphate (ATP) B) The loss of fluids due to decreased skin integrity and decreased stomach acids due to increased parasympathetic activity C) The release of catecholamines that creates an increase in metabolic rate and caloric requirements D) The increase in GI peristalsis during shock and the resulting diarrhea

C

The nurse caring for a patient with a spinal cord injury notes that the patient is exhibiting early signs and symptoms of disuse syndrome. Which of the following is the most appropriate nursing action? A) Limit the amount of assistance provided with ADLs. B) Collaborate with the physical therapist and immobilize the patient's extremities temporarily. C) Increase the frequency of ROM exercises. D) Educate the patient about the importance of frequent position changes

C

The nurse is admitting a patient with complaints of dyspnea on exertion and fatigue. The patients ECG shows dysrhythmias that are sometimes associated with left ventricular hypertrophy. What diagnostic tool would be most helpful in diagnosing cardiomyopathy? A) Cardiac catheterization B) Arterial blood gases C) Echocardiogram D) Exercise stress test

C

The nurse is assessing the patient for the presence of a Chvosteks sign. What electrolyte imbalance would a positive Chvosteks sign indicate? A) Hypermagnesemia B) Hyponatremia C) Hypocalcemia D) Hyperkalemia

C

The nurse is caring for a 39-year-old woman with a family history of breast cancer. She requested a breast tumor marking test and the results have come back positive. As a result, the patient is requesting a bilateral mastectomy. This surgery is an example of what type of oncologic surgery? A) Salvage surgery B) Palliative surgery C) Prophylactic surgery D) Reconstructive surgery

C

The nurse is caring for a 46-year-old patient recently diagnosed with the early stages of lung cancer. The nurse is aware that the preferred method of treating patients with nonsmall cell tumors is what? A) Chemotherapy B) Radiation C) Surgical resection D) Bronchoscopic opening of the airway

C

The nurse is caring for a patient in the ICU who has been diagnosed with multiple organ dysfunction syndrome (MODS). The nurses plan of care should include which of the following interventions? A) Encouraging the family to stay hopeful and educating them to the fact that, in nearly all cases, the prognosis is good B) Encouraging the family to leave the hospital and to take time for themselves as acute care of MODS patients may last for several months C) Promoting communication with the patient and family along with addressing end-of-life issues D) Discussing organ donation on a number of different occasions to allow the family time to adjust to the idea

C

The nurse is caring for a patient receiving hemodialysis three times weekly. The patient has had surgery to form an arteriovenous fistula. What is most important for the nurse to be aware of when providing care for this patient? A) Using a stethoscope for auscultating the fistula is contraindicated. B) The patient feels best immediately after the dialysis treatment. C) Taking a BP reading on the affected arm can damage the fistula. D) The patient should not feel pain during initiation of dialysis.

C

The nurse is caring for a patient who has central venous pressure (CVP) monitoring in place. The nurses most recent assessment reveals that CVP is 7 mm Hg. What is the nurses most appropriate action? A) Arrange for continuous cardiac monitoring and reposition the patient. B) Remove the CVP catheter and apply an occlusive dressing. C) Assess the patient for fluid overload and inform the physician. D) Raise the head of the patients bed and have the patient perform deep breathing exercise, if possible.

C

The nurse is caring for a patient who has had a dysrhythmic event. The nurse is aware of the need to assess for signs of diminished cardiac output (CO). What change in status may signal to the nurse a decrease in cardiac output? A) Increased blood pressure B) Bounding peripheral pulses C) Changes in level of consciousness D) Skin flushing

C

The nurse is caring for an 84-year-old man who has just returned from the OR after inguinal hernia repair. The OR report indicates that the patient received large volumes of IV fluids during surgery and the nurse recognizes that the patient is at risk for left-sided heart failure. What signs and symptoms would indicate left-sided heart failure? A) Jugular vein distention B) Right upper quadrant pain C) Bibasilar fine crackles D) Dependent edema

C

The nurse is creating an education plan for a patient who underwent a nephrectomy for the treatment of a renal tumor. What should the nurse include in the teaching plan? A) The importance of increased fluid intake B) Signs and symptoms of rejection C) Inspection and care of the incision D) Techniques for preventing metastasis

C

The nurse is discharging home a patient who suffered a stroke. He has a flaccid right arm and leg and is experiencing problems with urinary incontinence. The nurse makes a referral to a home health nurse because of an awareness of what common patient response to a change in body image? A) Denial B) Fear C) Depression D) Disassociation

C

The nurse is performing a spiritual assessment on a dying client. Which question provides the most accurate data on this aspect of the client's life? "Do you believe in God?" "Tell me about the history of religion in your life." "What gives you purpose and meaning in your life?" "Where have you been attending church for the past several years

C

The nurse is preparing health education for a patient who is being discharged after hospitalization for a hemorrhagic stroke. What content should the nurse include in this education? A) Mild, intermittent seizures can be expected. B) Take ibuprofen for complaints of a serious headache. C) Take antihypertensive medication as ordered. D) Drowsiness is normal for the first week after discharge.

C

The nurse is preparing to suction a patient with an endotracheal tube. What should be the nurses first step in the suctioning process? A) Explain the suctioning procedure to the patient and reposition the patient. B) Turn on suction source at a pressure not exceeding 120 mm Hg. C) Assess the patients lung sounds and SAO2 via pulse oximeter. D) Perform hand hygiene and don nonsterile gloves, goggles, gown, and mask.

C

The nurse is providing care for a patient who is in shock after massive blood loss from a workplace injury. The nurse recognizes that many of the findings from the most recent assessment are due to compensatory mechanisms. What is a compensatory mechanism to increase cardiac output during hypovolemic states? A) Third spacing of fluid B) Dysrhythmias C) Tachycardia D) Gastric hypermotility

C

The nurse is reviewing the medication administration record of a female patient who possesses numerous risk factors for stroke. Which of the woman's medications carries the greatest potential for reducing her risk of stroke? A) Naproxen 250 PO b.i.d. B) Calcium carbonate 1,000 mg PO b.i.d. C) Aspirin 81 mg PO o.d. D) Lorazepam 1 mg SL b.i.d. PRN

C

The nurse notes that a patient has developed a cough productive for mucoid sputum, is short of breath, has cyanotic hands, and has noisy, moist-sounding, rapid breathing. These symptoms and signs are suggestive of what health problem? A) Pericarditis B) Cardiomyopathy C) Pulmonary edema D) Right ventricular hypertrophy

C

The nurse on the pediatric unit is caring for a 10-year-old boy with a diagnosis of hemophilia. The nurse knows that a priority nursing diagnosis for a patient with hemophilia is what? A) Hypothermia B) Diarrhea C) Ineffective coping D) Imbalanced nutrition: Less than body requirements

C

The nursing educator is presenting a case study of an adult patient who has abnormal ventricular depolarization. This pathologic change would be most evident in what component of the ECG? A) P wave B) T wave C) QRS complex D) U wave

C

There has been a radiation-based terrorist attack and a patient is experiencing vomiting, diarrhea, and shock after the attack. How will the patient's likelihood of survival be characterized? A) Probable B) Possible C) Improbable D) Extended

C

Upon admission for symptoms of alcohol withdrawal, a client states, I havent eaten in 3 days. Assessment reveals BP 170/100 mm Hg, P 110, R 28, and T 97F (36C) with dry skin, dry mucous membranes, and poor skin turgor. What should be the priority nursing diagnosis? A. Knowledge deficit B. Fluid volume excess C. Imbalanced nutrition: less than body requirements D. Ineffective individual coping

C

What nursing action best demonstrates primary cancer prevention? A. Encouraging yearly Pap Tests B. Teaching testicular self examination C. Teaching clients to wear sunscreen D. Facilitating screening mamnograms

C

While assessing the patient, the nurse observes constant bubbling in the water-seal chamber of the patients closed chest-drainage system. What should the nurse conclude? A) The system is functioning normally. B) The patient has a pneumothorax. C) The system has an air leak. D) The chest tube is obstructed

C

While developing an emergency operations plan (EOP), the committee is discussing the components of the EOP. During the post-incident response of an emergency operations plan, what activity will take place? A) Deciding when the facility will go from disaster response to daily activities B) Conducting practice drills for the community and facility C) Conducting a critique and debriefing for all involved in the incident D) Replacing the resources in the facility

C

You are called to your patients room by a family member who voices concern about the patients status. On assessment, you find the patient tachypnic, lethargic, weak, and exhibiting a diminished cognitive ability. You also find 3+ pitting edema. What electrolyte imbalance is the most plausible cause of this patients signs and symptoms? A) Hypocalcemia B) Hyponatremia C) Hyperchloremia D) Hypophosphatemia

C

An 84-year-old woman diagnosed with cancer is admitted to the oncology unit for surgical treatment. The patient has been on chemotherapeutic agents to decrease the tumor size prior to the planned surgery. The nurse caring for the patient is aware that what precipitating factors in this patient may contribute to AKI? Select all that apply. A) Anxiety B) Low BMI C) Age-related physiologic changes D) Chronic systemic disease E) NPO status

C, D

A nurse is admitting a patient with immune thrombocytopenic purpura to the unit. In completing the admission assessment, the nurse must be alert for what medications that potentially alter platelet function? Select all that apply. A) Antihypertensives B) Penicillins C) Sulfa-containing medications D) Aspirin-based drugs E) NSAIDs

C,D,E

A patient is admitted to the orthopedic unit with a fractured femur after a motorcycle accident. The patient has been placed in traction until his femur can be rodded in surgery. For what early complications should the nurse monitor this patient? Select all that apply. A) Systemic infection B) Complex regional pain syndrome C) Deep vein thrombosis D) Compartment syndrome E) Fat embolism

C,D,E

One of the functions of nursing care of the terminally ill is to support the patient and his or her family as they come to terms with the diagnosis and progression of the disease process. How should nurses support patients and their families during this process? Select all that apply. A) Describe their personal experiences in dealing with end-of-life issues. B) Encourage the patient and family to ìkeep fightingî as a cure may come. C) Try to appreciate and understand the illness from the patient's perspective. D) Assist patients with performing a life review. E) Provide interventions that facilitate end-of-life closure.

C,D,E

The nurse educator is discussing neoplasms with a group of recent graduates. The educator explains that the effects of neoplasms are caused by the compression and infiltration of normal tissue. The physiologic changes that result can cause what pathophysiologic events? Select all that apply. A) Intracranial hemorrhage B) Infection of cerebrospinal fluid C) Increased ICP D) Focal neurologic signs E) Altered pituitary function

C,D,E

. A patient who has been exposed to anthrax is being treated in the local hospital. The nurse should prioritize what health assessments? A) Integumentary assessment B) Assessment for signs of hemorrhage C) Neurologic assessment D) Assessment of respiratory status

D

A 37-year-old man is brought to the clinic by his wife because he is experiencing loss of motor function and sensation. The physician suspects the patient has a spinal cord tumor and hospitalizes him for diagnostic testing. In light of the need to diagnose spinal cord compression from a tumor, the nurse will most likely prepare the patient for what test? A) Anterior-posterior x-ray B) Ultrasound C) Lumbar puncture D) MRI

D

A 54-year-old man has just been diagnosed with small cell lung cancer. The patient asks the nurse why the doctor is not offering surgery as a treatment for his cancer. What fact about lung cancer treatment should inform the nurses response? A) The cells in small cell cancer of the lung are not large enough to visualize in surgery. B) Small cell lung cancer is self-limiting in many patients and surgery should be delayed. C) Patients with small cell lung cancer are not normally stable enough to survive surgery. D) Small cell cancer of the lung grows rapidly and metastasizes early and extensively.

D

A client comes to a psychiatric clinic, experiencing sudden extreme fatigue and decreased sleep and appetite. The client works 12 hours a day and rates anxiety as 8/10 on a numeric scale. What correctly written long-term outcome is realistic in addressing this client's crisis? A. The client will change his or her type A personality traits to more adaptive ones by week 1. B. The client will list five positive self-attributes. C. The client will examine how childhood events led to an overachieving orientation. D. The client will return to previous adaptive levels of functioning by week 6

D

A client, who recently delivered a stillborn baby, has a diagnosis of adjustment disorder unspecified. The nurse case manager should expect which client presentation that is characteristic of this diagnosis? A. The client worries continually and appears nervous and jittery. B. The client complains of a depressed mood, is tearful, and feels hopeless. C. The client is belligerent, violates others' rights, and defaults on legal responsibilities. D. The client complains of many physical ailments, refuses to socialize, and quits her job.

D

A clinic patient is being treated for polycythemia vera and the nurse is providing health education. What practice should the nurse recommend in order to prevent the complications of this health problem? A) Avoiding natural sources of vitamin K B) Avoiding altitudes of 1500 feet (457 meters) C) Performing active range of motion exercises daily D) Avoiding tight and restrictive clothing on the legs

D

A critical care nurse is caring for a patient with a pulmonary artery catheter in place. What does this catheter measure that is particularly important in critically ill patients? A) Pulmonary artery systolic pressure B) Right ventricular afterload C) Pulmonary artery pressure D) Left ventricular preload

D

A critically ill patient is admitted to the ICU. The physician decides to use intra-arterial pressure monitoring. After this intervention is performed, what assessment should the nurse prioritize in the plan of care? A) Fluctuations in core body temperature B) Signs and symptoms of esophageal varices C) Signs and symptoms of compartment syndrome D) Perfusion distal to the insertion site

D

A group of nurses are participating in orientation to a telemetry unit. What should the staff educator tell this class about ST segments? A) They are the part of an ECG that reflects systole. B) They are the part of an ECG used to calculate ventricular rate and rhythm. C) They are the part of an ECG that reflects the time from ventricular depolarization through repolarization. D) They are the part of an ECG that represents early ventricular repolarization.

D

A highly agitated client paces the unit and states, "I could buy and sell this place." The client's mood fluctuates from fits of laughter to outbursts of anger. Which is the most accurate documentation of this client's behavior? A. "Rates mood 8/10. Exhibiting looseness of association. Euphoric." B. "Mood euthymic. Exhibiting magical thinking. Restless." C. "Mood labile. Exhibiting delusions of reference. Hyperactive." D. "Agitated and pacing. Exhibiting grandiosity. Mood labile." D. "Agitated and pacing. Exhibiting grandiosity. Mood labile."

D

A local public health nurse is informed that a cook in a local restaurant has been diagnosed with hepatitis A. What should the nurse advise individuals to obtain who ate at this restaurant and have never received the hepatitis A vaccine? A. The hepatitis A vaccine B. Albumin infusion C. The hepatitis A and B vaccines D. An immune globulin injection

D

A mother is concerned about her ability to perform in her new role. She is quite anxious and refuses to leave the postpartum unit. To offer effective client care, a nurse should recognize which information about this type of crisis? A. This type of crisis is precipitated by unexpected external stressors. B. This type of crisis is precipitated by preexisting psychopathology. C. This type of crisis is precipitated by an acute response to an external situational stressor. D. This type of crisis is precipitated by normal life-cycle transitions that overwhelm the client.

D

A nurse has made a referral to a grief support group, knowing that many individuals find these both comforting and beneficial after the death of a loved one. What is the most important accomplishment available by attending a grief support group? A) Providing a framework for incorporating the old life into the new life B) Normalizing adaptation to a continuation of the old life C) Aiding in adjusting to using old, familiar social skills D) Normalization of feelings and experiences

D

A nurse holds the hand of a client who is withdrawing from alcohol. What is the nurses rationale for this intervention? A. To assess for emotional strength B. To assess for Wernicke-Korsakoff syndrome C. To assess for tachycardia D. To assess for fine tremors

D

A nurse is caring for a patient who has sustained a deep partial-thickness burn injury. In prioritizing the nursing diagnoses for the plan of care, the nurse will give the highest priority to what nursing diagnosis? A) Activity Intolerance B) Anxiety C) Ineffective Coping D) Acute Pain

D

A nurse is caring for a patient with liver failure and is performing an assessment in the knowledge of the patient's increased risk of bleeding. The nurse recognizes that this risk is related to the patient's inability to synthesize prothrombin in the liver. What factor most likely contributes to this loss of function? A. Alterations in glucose metabolism B. Retention of bile salts C. Inadequate production of albumin by hepatocytes D. Inability of the liver to use vitamin K.

D

A nurse is performing an admission assessment for an 81-year-old patient who generally enjoys good health. When considering normal, age-related changes to hepatic function, the nurse should anticipate what finding? A. Similar liver size and texture as in younger adults B. A nonpalpable liver C. A slightly enlarged liver with palpably hard edges D. A slightly decreased size of the liver

D

A nurse is planning the care of a patient who has a diagnosis of hemophilia A. When addressing the nursing diagnosis of Acute Pain Related to Joint Hemorrhage, what principle should guide the nurses choice of interventions? A) Gabapentin (Neurontin) is effective because of the neuropathic nature of the patients pain. B) Opioids partially inhibit the patients synthesis of clotting factors. C) Opioids may cause vasodilation and exacerbate bleeding. D) NSAIDs are contraindicated due to the risk for bleeding.

D

A nurse is preparing to discharge a patient from the emergency department after receiving treatment for an ankle sprain. While providing discharge education, the nurse should encourage which of the following? A) Apply heat for the first 24 to 48 hours after the injury. B) Maintain the ankle in a dependent position. C) Exercise hourly by performing rotation exercises of the ankle. D) Keep an elastic compression bandage on the ankle.

D

A nurse is preparing to discharge an emergency department patient who has been fitted with a sling to support her arm after a clavicle fracture. What should the nurse instruct the patient to do? A) Elevate the arm above the shoulder 3 to 4 times daily. B) Avoid moving the elbow, wrist, and fingers until bone remodeling is complete. C) Engage in active range of motion using the affected arm. D) Use the arm for light activities within the range of motion.

D

A nurse is providing a class on osteoporosis at the local seniors' center. Which of the following statements related to osteoporosis is most accurate? A) Osteoporosis is categorized as a disease of the elderly. B) A nonmodifiable risk factor for osteoporosis is a person's level of activity. C) Secondary osteoporosis occurs in women after menopause. D) Slow discontinuation of corticosteroid therapy can halt the progression of the osteoporosis.

D

A nurse is reviewing the care of a patient who has a long history of lower back pain that has not responded to conservative treatment measures. The nurse should anticipate the administration of what drug? A) Calcitonin B) Prednisone C) Aspirin D) Cyclobenzaprine

D

A nurse understands that when a practitioner corrects subluxation by manipulating the vertebrae of the spinal column the practitioner is employing which therapy? A. Allopathic therapy B. Therapeutic touch therapy C. Massage therapy D. Chiropractic therapy

D

A nursing student is writing a care plan for a newly admitted patient who has been diagnosed with a stroke. What major nursing diagnosis should most likely be included in the patient's plan of care? A) Adult failure to thrive B) Post-trauma syndrome C) Hyperthermia D) Disturbed sensory perception

D

A patient arrives in the emergency department after being burned in a house fire. The patients burns cover the face and the left forearm. What extent of burns does the patient most likely have? A) 13% B) 25% C) 9% D) 18%

D

A patient calls his cardiologists office and talks to the nurse. He is concerned because he feels he is being defibrillated too often. The nurse tells the patient to come to the office to be evaluated because the nurse knows that the most frequent complication of ICD therapy is what? A) Infection B) Failure to capture C) Premature battery depletion D) Oversensing of dysrhythmias

D

A patient has been brought to the ED after suffering genitourinary trauma in an assault. Initial assessment reveals that the patient's bladder is distended. What is the nurse's most appropriate action? A) Withhold fluids from the patient. B) Perform intermittent urinary catheterization. C) Insert a narrow-gauge indwelling urinary catheter. D) Await orders following the urologist's assessment.

D

A patient is being treated for DIC and the nurse has prioritized the nursing diagnosis of Risk for Deficient Fluid Volume Related to Bleeding. How can the nurse best determine if goals of care relating to this diagnosis are being met? A) Assess for edema. B) Assess skin integrity frequently. C) Assess the patients level of consciousness frequently. D) Closely monitor intake and output

D

A patient is being treated on the acute medical unit for acute pancreatitis. The nurse has identified a diagnosis of Ineffective Breathing Pattern Related to Pain. What intervention should the nurse perform in order to best address this diagnosis? A) Position the patient supine to facilitate diaphragm movement. B) Administer corticosteroids by nebulizer as ordered. C) Perform oral suctioning as needed to remove secretions. D) Maintain the patient in a semi-Fowler's position whenever possible.

D

A patient is brought to the ED by paramedics, who report that the patient has partial-thickness burns on the chest and legs. The patient has also suffered smoke inhalation. What is the priority in the care of a patient who has been burned and suffered smoke inhalation? A) Pain B) Fluid balance C) Anxiety and fear D) Airway management

D

A patient is brought to the ER in an unconscious state. The physician notes that the patient is in need of emergency surgery. No family members are present, and the patient does not have identification. What action by the nurse is most important regarding consent for treatment? A) Ask the social worker to come and sign the consent. B) Contact the police to obtain the patient's identity. C) Obtain a court order to treat the patient. D) Clearly document LOC and health status on the patient's chart.

D

A patient presents at the clinic with complaints of morning numbness, cramping, and stiffness in his fourth and fifth fingers. What disease process should the nurse suspect? A) Tendonitis B) A ganglion C) Carpal tunnel syndrome D) Dupuytren's disease

D

A patient recovering from a stroke has severe shoulder pain from subluxation of the shoulder and is being cared for on the unit. To prevent further injury and pain, the nurse caring for this patient is aware of what principle of care? A) The patient should be fitted with a cast because use of a sling should be avoided due to adduction of the affected shoulder. B) Elevation of the arm and hand can lead to further complications associated with edema. C) Passively exercising the affected extremity is avoided in order to minimize pain. D) The patient should be taught to interlace fingers, place palms together, and slowly bring scapulae forward to avoid excessive force to shoulder.

D

A patient who attempted suicide being treated in the ED is accompanied by his mother, father, and brother. When planning the nursing care of this family, the nurse should perform which of the following action? A) Refer the family to psychiatry in order to provide them with support. B) Explore the causes of the patient's suicide attempt with the family. C) Encourage the family to participate in the bedside care of the patient. D) Ensure that the family receives appropriate crisis intervention services

D

A patient who has recently recovered from a systemic viral infection is undergoing diagnostic testing for myocarditis. Which of the nurses assessment findings is most consistent with myocarditis? A) Sudden changes in level of consciousness (LOC) B) Peripheral edema and pulmonary edema C) Pleuritic chest pain D) Flulike symptoms

D

A patient who suffered a spinal cord injury is experiencing an exaggerated autonomic response. What aspect of the patient's current health status is most likely to have precipitated this event? A) The patient received a blood transfusion. B) The patient's analgesia regimen was recent changed. C) The patient was not repositioned during the night shift. D) The patient's urinary catheter became occluded

D

A patient with Von Willebrand disease (vWD) has experienced recent changes in bowel function that suggest the need for a screening colonoscopy. What intervention should be performed in anticipation of this procedure? A) The patient should not undergo the normal bowel cleansing protocol prior to the procedure. B) The patient should receive a unit of fresh-frozen plasma 48 hours before the procedure. C) The patient should be admitted to the surgical unit on the day before the procedure. D) The patient should be given necessary clotting factors before the procedure.

D

A patient with a cerebral aneurysm exhibits signs and symptoms of an increase in intracranial pressure (ICP). What nursing intervention would be most appropriate for this patient? A) Range-of-motion exercises to prevent contractures B) Encouraging independence with ADLs to promote recovery C) Early initiation of physical therapy D) Absolute bed rest in a quiet, nonstimulating environment

D

A patient with a pulmonary embolism is being treated with a heparin infusion. What diagnostic finding suggests to the nurse that treatment is effective? A) The patients PT is within reference ranges. B) Arterial blood sampling tests positive for the presence of factor XIII. C) The patients platelet level is below 100,000/mm3. D) The patients activated partial thromboplastin time (aPTT) is 1.5 to 2.5 times the control value.

D

A patient with chronic kidney disease has been hospitalized and is receiving hemodialysis on a scheduled basis. The nurse should include which of the following actions in the plan of care? A) Ensure that the patient moves the extremity with the vascular access site as little as possible. B) Change the dressing over the vascular access site at least every 12 hours. C) Utilize the vascular access site for infusion of IV fluids. D) Assess for a thrill or bruit over the vascular access site each shift.

D

A patient with liver cancer is being discharged home with a hepatic artery catheter in place. The nurse should be aware that this catheter will facilitate which of the following? A. Continuous monitoring for portal hypertension B. Administration of immunosuppressive drugs during the first weeks after transplantation C. Real-time monitoring of vascular changes in the hepatic system D. Delivery of a continuous chemotherapeutic dose

D

A patients burns are estimated at 36% of total body surface area; fluid resuscitation has been ordered in the emergency department. After establishing intravenous access, the nurse should anticipate the administration of what fluid? A) 0.45% NaCl with 20 mEq/L KCl B) 0.45% NaCl with 40 mEq/L KCl C) Normal saline D) Lactated Ringers

D

A patients most recent laboratory results show a slight decrease in potassium. The physician has opted to forego drug therapy but has suggested increasing the patients dietary intake of potassium. Which of the following would be a good source of potassium? A) Apples B) Asparagus C) Carrots D) Bananas

D

An intensive care nurse is aware of the need to identify patients who may be at risk of developing disseminated intravascular coagulation (DIC). Which of the following ICU patients most likely faces the highest risk of DIC? A) A patient with extensive burns B) A patient who has a diagnosis of acute respiratory distress syndrome C) A patient who suffered multiple trauma in a workplace accident D) A patient who is being treated for septic shock

D

An interdisciplinary team has been commissioned to create policies and procedures aimed at preventing acute hemolytic transfusion reactions. What action has the greatest potential to reduce the risk of this transfusion reaction? A) Ensure that blood components are never infused at a rate greater than 125 mL/hr. B) Administer prophylactic antihistamines prior to all blood transfusions. C) Establish baseline vital signs for all patients receiving transfusions. D) Be vigilant in identifying the patient and the blood component

D

An involuntarily committed client, when offered a dinner tray, pushes it off the bedside table onto the floor. Which intervention should a nurse prioritize to address this behavior? A. Initiate forced medication protocol. B. Help the client to explore the source of anger. C. Ignore the act to avoid reinforcing the behavior. D. With staff support and a show of solidarity, set firm limits on the behavior.

D

An obtunded patient is admitted to the ED after ingesting bleach. The nurse should prepare to assist with what intervention? A) Prompt administration of an antidote B) Gastric lavage C) Administration of activated charcoal D) Helping the patient drink large amounts of water

D

An occupational health nurse is called to the floor of a factory where a worker has sustained a flash burn to the right arm. The nurse arrives and the flames have been extinguished. The next step is to cool the burn. How should the nurse cool the burn? A) Apply ice to the site of the burn for 5 to 10 minutes. B) Wrap the patients affected extremity in ice until help arrives. C) Apply an oil-based substance or butter to the burned area until help arrives. D) Wrap cool towels around the affected extremity intermittently.

D

An older adult patient sought care for the treatment of a swollen, painful knee joint. Diagnostic imaging and culturing of synovial fluid resulted in a diagnosis of septic arthritis. The nurse should prioritize which of the following aspects of care? A) Administration of oral and IV corticosteroids as ordered B) Prevention of falls and pathologic fractures C) Maintenance of adequate serum levels of vitamin D D) Intravenous administration of antibiotics

D

An oncology nurse is contributing to the care of a patient who has failed to respond appreciably to conventional cancer treatments. As a result, the care team is considering the possible use of biologic response modifiers (BRFs). The nurse should know that these achieve a therapeutic effect by what means? A) Promoting the synthesis and release of leukocytes B) Focusing the patient's immune system exclusively on the tumor C) Potentiating the effects of chemotherapeutic agents and radiation therapy D) Altering the immunologic relationship between the tumor and the patient

D

An orthopedic nurse is caring for a patient who is postoperative day one following foot surgery. What nursing intervention should be included in the patient's subsequent care? A) Dressing changes should not be performed unless there are clear signs of infection. B) The surgical site can be soaked in warm bath water for up to 5 minutes. C) The surgical site should be cleansed with hydrogen peroxide once daily. D) The foot should be elevated in order to prevent edema.

D

Diagnostic testing has resulted in a diagnosis of acute myeloid leukemia (AML) in an adult patient who is otherwise healthy. The patient and the care team have collaborated and the patient will soon begin induction therapy. The nurse should prepare the patient for which of the following? A) Daily treatment with targeted therapy medications B) Radiation therapy on a daily basis C) Hematopoietic stem cell transplantation D) An aggressive course of chemotherapy

D

Eye movement desensitization and reprocessing (EMDR) has been empirically validated for which disorder? A. Adjustment disorder B. Generalized anxiety disorder C. Panic disorder D. Post-traumatic stress disorder

D

Following diagnostic testing, a patient has been admitted to the ICU and placed on cerebral aneurysm precautions. What nursing action should be included in patient's plan of care? A) Supervise the patient's activities of daily living closely. B) Initiate early ambulation to prevent complications of immobility. C) Provide a high-calorie, low-protein diet. D) Perform all of the patient's hygiene and feeding.

D

In all types of shock, nutritional demands increase rapidly as the body depletes its stores of glycogen. Enteral nutrition is the preferred method of meeting these increasing energy demands. What is the basis for enteral nutrition being the preferred method of meeting the bodys needs? A) It slows the proliferation of bacteria and viruses during shock. B) It decreases the energy expended through the functioning of the GI system. C) It assists in expanding the intravascular volume of the body. D) It promotes GI function through direct exposure to nutrients.

D

Radiographs of a boy's upper arm show that the humerus appears to be fractured on one side and slightly bent on the other. This diagnostic result suggests what type of fracture? A) Impacted B) Compound C) Compression D) Greenstick

D

Sepsis is an evolving process, with neither clearly definable clinical signs and symptoms nor predictable progression. As the ICU nurse caring for a patient with sepsis, the nurse knows that tissue perfusion declines during sepsis and the patient begins to show signs of organ dysfunction. What sign would indicate to the nurse that end-organ damage may be occurring? A) Urinary output increases B) Skin becomes warm and dry C) Adventitious lung sounds occur in the upper airway D) Heart and respiratory rates are elevated

D

The cardiac monitor alarm alerts the critical care nurse that the patient is showing no cardiac rhythm on the monitor. The nurses rapid assessment suggests cardiac arrest. In providing cardiac resuscitation documentation, how will the nurse describe this initial absence of cardiac rhythm? A) Pulseless electrical activity (PEA) B) Ventricular fibrillation C) Ventricular tachycardia D) Asystole

D

The emergency nurse is admitting a patient experiencing a GI bleed who is believed to be in the compensatory stage of shock. What assessment finding would be most consistent with the early stage of compensation? A) Increased urine output B) Decreased heart rate C) Hyperactive bowel sounds D) Cool, clammy skin

D

The hospice nurse is caring for a 45-year-old mother of three young children in the patient's home. During the most recent visit, the nurse has observed that the patient has a new onset of altered mental status, likely resulting from recently diagnosed brain metastases. What goal of nursing interventions should the nurse identify? A) Helping the family to understand why the patient needs to be sedated B) Making arrangements to promptly move the patient to an acute-care facility C) Explaining to the family that death is near and the patient needs around-the-clock nursing care D) Teaching family members how to interact with, and ensure safety for, the patient with impaired cognitio

D

The nurse is caring for a patient with increased intracranial pressure (ICP). The patient has a nursing diagnosis of "ineffective cerebral tissue perfusion." What would be an expected outcome that the nurse would document for this diagnosis? A) Copes with sensory deprivation. B) Registers normal body temperature. C) Pays attention to grooming. D) Obeys commands with appropriate motor responses

D

What should be included in the patient's care plan when establishing an exercise program for a patient affected by a stroke? A) Schedule passive range of motion every other day. B) Keep activity limited, as the patient may be over stimulated. C) Have the patient perform active range-of-motion (ROM) exercises once a day. D) Exercise the affected extremities passively four or five times a day.

D

When circulatory shock occurs, there is massive vasodilation causing pooling of the blood in the periphery of the body. An ICU nurse caring for a patient in circulatory shock should know that the pooling of blood in the periphery leads to what pathophysiological effect? A) Increased stroke volume B) Increased cardiac output C) Decreased heart rate D) Decreased venous return

D

Which client statement indicates a knowledge deficit related to substance use? A. Although its legal, alcohol is one of the most widely abused drugs in our society. B. Tolerance to heroin develops quickly. C. Flashbacks from LSD use may reoccur spontaneously. D. Marijuana is like smoking cigarettes. Everyone does it. Its essentially harmless.

D

Which medication orders should a nurse anticipate for a client who has a history of complicated withdrawal from benzodiazepines? A. Haloperidol (Haldol) and fluoxetine (Prozac) B. Carbamazepine (Tegretol) and donepezil (Aricept) C. Disulfiram (Antabuse) and lorazepam (Ativan) D. Chlordiazepoxide (Librium) and phenytoin (Dilantin)

D

Which term should a nurse use to describe the administration of a central nervous system (CNS) depressant during alcohol withdrawal? A. Antagonist therapy B. Deterrent therapy C. Codependency therapy D. Substitution therapy

D

You are an emergency-room nurse caring for a trauma patient. Your patient has the following arterial blood gas results: pH 7.26, PaCO2 28, HCO3 11 mEq/L. How would you interpret these results? A) Respiratory acidosis with no compensation B) Metabolic alkalosis with a compensatory alkalosis C) Metabolic acidosis with no compensation D) Metabolic acidosis with a compensatory respiratory alkalosis

D

You are caring for a patient who is being treated on the oncology unit with a diagnosis of lung cancer with bone metastases. During your assessment, you note the patient complains of a new onset of weakness with abdominal pain. Further assessment suggests that the patient likely has a fluid volume deficit. You should recognize that this patient may be experiencing what electrolyte imbalance? A) Hypernatremia B) Hypomagnesemia C) Hypophosphatemia D) Hypercalcemia

D

You are caring for a patient with a secondary diagnosis of hypermagnesemia. What assessment finding would be most consistent with this diagnosis? A) Hypertension B) Kussmaul respirations C) Increased DTRs D) Shallow respirations

D

You are working on a burns unit and one of your acutely ill patients is exhibiting signs and symptoms of third spacing. Based on this change in status, you should expect the patient to exhibit signs and symptoms of what imbalance? A) Metabolic alkalosis B) Hypermagnesemia C) Hypercalcemia D) Hypovolemia

D

client asks a nurse to explain the difference between complementary and alternative medicine. Which is an appropriate nursing reply? A. "Alternative medicine is a more acceptable practice than complementary medicine." B. "Alternative and complementary medicine are terms that essentially mean the same thing." C. "Complementary medicine disregards traditional medical approaches." D. "Complementary therapies partner alternative medicine with traditional medical practice."

D

A nurse is providing an educational class to a group of older adults at a community senior center. In an effort to prevent osteoporosis, the nurse should encourage participants to ensure that they consume the recommended adequate intake of what nutrients? Select all that apply. A) Vitamin B12 B) Potassium C) Calcitonin D) Calcium E) Vitamin D

D,E

A client diagnosed with borderline personality disorder brings up a conflict with the staff in a community meeting and develops a following of clients who unreasonably demand modification of unit rules. How can the nursing staff best handle this situation? A. Allow the clients to apply the democratic process when developing unit rules. B. Maintain consistency of care by open communication to avoid staff manipulation. C. Allow the client spokesman to verbalize concerns during a unit staff meeting. D. Maintain unit order by the application of autocratic leadership.

b

A nurse is caring for a patient who has been scheduled for endoscopic retrograde cholangiopancreatography (ERCP) the following day. When providing anticipatory guidance for this patient, the nurse should describe what aspect of this diagnostic procedure? A) The need to protect the incision postprocedure B) The use of moderate sedation C) The need to infuse 50% dextrose during the procedure D) The use of general anesthesia

b

A nurse is preparing a plan of care for a patient with pancreatic cysts that have necessitated drainage through the abdominal wall. What nursing diagnosis should the nurse prioritize? A) Disturbed Body Image B) Impaired Skin Integrity C) Nausea D) Risk for Deficient Fluid Volume

b

A patient is brought to the trauma center by ambulance after sustaining a high cervical spinal cord injury 1½ hours ago. Endotracheal intubation has been deemed necessary and the nurse is preparing to assist. What nursing diagnosis should the nurse associate with this procedure? A) Risk for impaired skin integrity B) Risk for injury C) Risk for autonomic dysreflexia D)

b

During an assessment interview, a client diagnosed with antisocial personality disorder spits, curses, and refuses to answer questions. Which is the appropriate nursing response to this behavior? A. "You are very disrespectful. You need to learn to control yourself." B. "I understand that you are angry, but this behavior will not be tolerated." C. "What behaviors could you modify to improve this situation?" D. "What anti-personality disorder medications have helped you in the past?"

b

The nurse is caring for an acutely ill patient who has central venous pressure monitoring in place. What intervention should be included in the care plan of a patient with CVP in place? A) Apply antibiotic ointment to the insertion site twice daily. B) Change the site dressing whenever it becomes visibly soiled. C) Perform passive range-of-motion exercises to prevent venous stasis. D) Aspirate blood from the device once daily to test pH.

b

A client diagnosed with antisocial personality disorder comes to a nurses' station at 11:00 p.m. requesting to phone a lawyer to discuss filing for a divorce. The unit rules state that no phone calls are permitted after 10:00 p.m. Which nursing response is most appropriate? A. "Go ahead and use the phone. I know this pending divorce is stressful." B. "You know better than to break the rules. I'm surprised at you." C. "It is after the 10:00 p.m. phone curfew. You will be able to call tomorrow." D. "The decision to divorce should not be considered until you have a good night of sleep

c

A client diagnosed with paranoid personality disorder becomes violent on a unit. Which nursing intervention is most appropriate? A. Provide objective evidence, that violence is unwarranted. B. Initially restrain the client to maintain safety. C. Use clear, calm statements and a confident physical stance. D. Empathize with the client's paranoid perceptions.

c

A patient is being admitted to the neurologic ICU following an acute head injury that has resulted in cerebral edema. When planning this patients care, the nurse would expect to administer what priority medication? A) Hydrochlorothiazide (HydroDIURIL) B) Furosemide (Lasix) C) Mannitol (Osmitrol) D) Spirolactone (Aldactone)

c

A patient who had surgery for gallbladder disease has just returned to the postsurgical unit from postanesthetic recovery. The nurse caring for this patient knows to immediately report what assessment finding to the physician? A) Decreased breath sounds B) Drainage of bile-colored fluid onto the abdominal dressing C) Rigidity of the abdomen D) Acute pain with movement

c

A patient who has been on long-term phenytoin (Dilantin) therapy is admitted to the unit. In light of the adverse of effects of this medication, the nurse should prioritize which of the following in the patients plan of care? A) Monitoring of pulse oximetry B) Administration of a low-protein diet C) Administration of thorough oral hygiene D) Fluid restriction as ordered

c

A patient with liver disease has developed jaundice; the nurse is collaborating with the patient to develop a nutritional plan. The nurse should prioritize which of the following in the patient's plan? A. Increased potassium intake B. Fluid restriction to 2L per day C. Reduction in sodium intake D. High-protein, low-fat diet

c

A patient's low hemoglobin level has necessitated transfusion of PRBCs. Prior to administration, what action should the nurse perform? A) Have the patient identify his or her blood type in writing. B) Ensure that the patient has granted verbal consent for transfusion. C) Assess the patient's vital signs to establish baselines. D) Facilitate insertion of a central venous catheter

c

The ED nurse is caring for a patient who has been brought in by ambulance after sustaining a fall at home. What physical assessment finding is suggestive of a basilar skull fracture? A) Epistaxis B) Periorbital edema C) Bruising over the mastoid D) Unilateral facial numbness Ans:

c

A nurse tells a client that the nursing staff will start alternating weekend shifts. Which response should a nurse identify as characteristic of clients diagnosed with obsessive-compulsive personality disorder? A. "You really don't have to go by that schedule. I'd just stay home sick." B. "There has got to be a hidden agenda behind this schedule change." C. "Who do you think you are? I expect to interact with the same nurse every Saturday." D. "You can't make these kinds of changes! Isn't there a rule that governs this decision?"

d

A patient presents to the emergency department (ED) complaining of severe right upper quadrant pain. The patient states that his family doctor told him he had gallstones. The ED nurse should recognize what possible complication of gallstones? A) Acute pancreatitis B) Atrophy of the gallbladder C) Gallbladder cancer D) Gangrene of the gallbladder

d

Which nursing diagnosis should a nurse identify as appropriate when working with a client diagnosed with schizoid personality disorder? A. Altered thought processes R/T increased stress B. Risk for suicide R/T loneliness C. Risk for violence: directed toward others R/T paranoid thinking D. Social isolation R/T inability to relate to others

d


Kaugnay na mga set ng pag-aaral

Cambridge English/Compact First for School Second Edition/ Unit 1

View Set

revenue, expenses, assets, liabilities, Owner's Equity , or neither : unit 7 accounting test 12.1.17

View Set

APUSH Midterm Review (Chapters 1-23)

View Set

Lab activity 3 regional terminology

View Set

Chapter 7 DNA repair and mutations

View Set

Week 1 - Introduction to Organizational Behavior

View Set